You are on page 1of 125

[ wu :: riddles(easy) ]

relatively easy
you are a prisoner in a foreign land. your fate will be determined by a little game. there are two jars, one with 50 white marbles, and one with 50 black marbles. at this point, you are allowed to redistribute the marbles however you wish (e.g. swap a black marble with a white marble, etc.): the only requirement is that after you are done with the redistribution, every marble must be in one of the two jars. afterwards, both jars will be shaken up, and you will be blindfolded and presented

MARBLE JARS I

with one of the jars at random. then you pick one marble out of the jar given to you. if the marble you pull out is white, you live; if black, you die. how should you redistribute the marbles to maximize the probability that you live; what is this maximum probability (roughly)? Hint: Redundancy.

An Arab sheikh is old and must will his fortune to one of his two sons. He makes a proposition. His two sons will ride their camels in a race, and whichever camel crosses the finish line last will win the fortune for its owner. During the race, the two brothers wander aimlessly for days, neither willing to cross the finish line. In desperation, they ask a wise man for advice. He tells them something; then the brothers leap onto the camels and charge toward the finish line. What did the wise man say?

ARAB SHEIKH CAMELS

There are 3 black hats and 2 white hats in a box. Three men (we will call them A, B, & C) each reach into the box and place one of the hats on his own head. They cannot see what color hat they have chosen. The men are situated in a way that A can see the hats on B & C's heads, B can only see the hat on C's head and C cannot see any hats. When A is asked if he knows the color of the hat he is wearing, he says no. When B is asked if he knows the color of the hat he is wearing he says no. When C is asked if he knows the color of the hat he is wearing he says yes and he is correct. What color hat and how can this be?

3 HATS

One train leaves Los Angeles at 15mph heading for New York. Another train leaves from New York at 20mph heading for Los Angeles on the same track. The

HUMMINGBIRD

distance between LA and NY is about 5000 miles. If a bird, flying at 25mph, leaves from Los Angeles at the same time as the train and flies back and forth between the two trains until they collide, how far will the bird have traveled?

file:///C|/Documents%20and%20Settings/Administrator/M...20Documents/My%20Notepad/Jayesh%20placment/easy.shtml (2 of 28)8/7/2008 3:24:41 PM

[ wu :: riddles(easy) ]

scientific studies have shown that there is a direct, positive correlation between

FOOT SIZE IMPLIES SPELLING ABILITY

foot size and performance in spelling bees / spelling tests. how can you explain this correlation? Hint: Don't think too hard ... it's just for the most obvious reasons.

you have two ropes, each of which takes one hour to burn completely. both of these ropes are nonhomogeneous in thickness, meaning that some parts of the ropes are chunkier than other parts of the rope. using these nonhomogeneous ropes and a lighter, time 45 minutes. Note: Some clarification on what is meant by nonhomogeneous. For instance, maybe a particular section of rope that is 1/8 of the total length is really chunky, and takes 50 minutes to burn off. then it would take 10 minutes to burn off the remaning 7/8, since we know that the whole rope takes an hour to burn off. that's just an example; we don't know any such ratios beforehand. The point is, if you look at one of your ropes and cut it into pieces, you have no clue how long any individual piece will take to burn off.

NONHOMOGENEOUS ROPE BURNING

willywutang is hanging out on a heavily forested island that's really narrow: it's a narrow strip of land that's ten miles long. let's label one end of the strip A, and the other end B. a fire has started at A, and the fire is moving toward B at the rate of 1 mph. at the same time, there's a 2 mph wind blowing in the direction from A toward B. what can willywu do to save himself from burning to death?! assume that willywu can't swim and there are no boats, jetcopters, teleportation devices, etc.. (if he does nothing, willywu will be toast after at most 10 hours, since 10 miles / 1 mph = 10 hours) Forum thread: click here

WILLYWUTANG AND THE BURNING ISLAND OF DOOM

i flip a penny and a dime and hide the result from you. "at least one of the coins came up heads", i announce. what is the chance that both coins came up heads?

TWO COIN FLIPS

Hint: Think again; conditional probability is often very nonintuitive. Write out a table of possibilities.

file:///C|/Documents%20and%20Settings/Administrator/M...20Documents/My%20Notepad/Jayesh%20placment/easy.shtml (3 of 28)8/7/2008 3:24:41 PM

[ wu :: riddles(easy) ]

you have 20 coin machines, each of which produce the same kind of coin. you know how much a coin is supposed to weigh. one of the machines is defective, in that every coin it produces weighs 1 ounce less than it is supposed to. you also have an electronic weighing machine. how can you determine which of the 20 machines is defective with only one weighing? (by one use, we mean you put a bunch of stuff on the machine and read a number, and that's it -- you not allowed to accumulate weight onto the machine and watch the numbers ascend, because that's just like multiple weighings). you are allowed to crank out as many coins from each machine as you like.

COIN MACHINE WEIGHING

HOURGLASSES

You have two hourglasses: a 7 minute one and an 11 minute one. Using just these hourglasses, accurately time 15 minutes.

You are an archaeologist that has just unearthed a long-sought pair of ancient treasure chests. One chest is plated with silver, and the other is plated with gold. According to legend, one of the two chests is filled with great treasure, whereas the other chest houses a man-eating python that can rip your head off. Faced with a dilemma, you then notice that there are inscriptions on the chests: Silver Chest This chest contains the python. Gold Chest One of these two inscriptions is true.

LOGICAL SIGNS I

Based on these inscriptions, which chest should you open? Hint: argumentum ad ignorantiam. Thanks to Peter Surda for e-mailing me his unconventional analysis.

You are an archaeologist that has just unearthed a long-sought triplet of ancient treasure chests. One chest is plated with silver, one with gold, and one with bronze. According to legend, one of the three chests is filled with great treasure, whereas the other two chests both house man-eating pythons that can rip your head off. Faced with a dilemma, you then notice that there are inscriptions on the chests:

LOGICAL SIGNS II

Silver Chest Treasure is in this Chest.

Gold Chest Treasure is not in this Chest.

Bronze Chest Treasure is not in the Gold Chest.

You know that at least one of the inscriptions is true, and at least one of the inscriptions is false. Which chest do you open?

file:///C|/Documents%20and%20Settings/Administrator/M...20Documents/My%20Notepad/Jayesh%20placment/easy.shtml (4 of 28)8/7/2008 3:24:41 PM

[ wu :: riddles(easy) ]

Green numbers indicate how many pieces could move to that square on the next move. Blue squares show the possible locations of the following five different chess pieces:

How are the five pieces arranged?

CHESS PUZZLE I C

Green numbers indicate how many pieces could move to that square on the next move. Blue squares show the possible locations of the following five different chess pieces:

How are the five pieces arranged?

CHESS PUZZLE II C

file:///C|/Documents%20and%20Settings/Administrator/M...20Documents/My%20Notepad/Jayesh%20placment/easy.shtml (5 of 28)8/7/2008 3:24:41 PM

[ wu :: riddles(easy) ]

If you were to put a coin into an empty bottle and then insert a cork in the bottle's opening, how could you remove the coin without taking out the cork or breaking the bottle? Hint: Actually, people solve this riddle everyday. Let's say you're opening a wine bottle, and in the process you break the cork. What's the only thing left to do?

CORK, BOTTLE, COIN

Speaker: "Brothers and Sisters, I have none. But this man's Father is my Father's son."

FAMILY RELATIONS
Who is the speaker talking about?

ANALOG CLOCK I

An analog clock reads 3:15. What is the angle between the minute hand and hour hand?

Imagine an analog clock set to 12 o'clock. Note that the hour and minute hands

ANALOG CLOCK II

overlap. How many times each day do both the hour and minute hands overlap? How would you determine the exact times of the day that this occurs?

There are three closed and opaque cardboard boxes. One is labeled "APPLES", another is labeled "ORANGES", and the last is labeled "APPLES AND ORANGES". You know that the labels are currently misarranged, such that no box is correctly labeled. You would like to correctly rearrange these labels. To accomplish this, you may draw only one fruit from one of the boxes. Which box do you choose, and how do you then proceed to rearrange the labels? Note: (1/19/2003 1:23AM) Edited to add that the boxes are opaque.

APPLES AND ORANGES

What is the beginning of eternity, the end of time and space, the start of every

STUPID

end, and the end of every race?

A snail is at the bottom of a well that is 20 meters in depth. Every day the snail

CLIMBING SNAIL

climbs 5 meters upwards, but at night it slides 4 meters back downwards. How many days must elapse till the snail reaches the top of the well?

file:///C|/Documents%20and%20Settings/Administrator/M...20Documents/My%20Notepad/Jayesh%20placment/easy.shtml (6 of 28)8/7/2008 3:24:41 PM

[ wu :: riddles(easy) ]

You have a round birthday cake. With three straight slices of a knife, divide the

8-WAY CAKE SLICE

cake into 8 equal pieces. I know of two different solutions.

How many squares are on a chessboard (8 x 8)?

CHESSBOARD SQUARE COUNT

Followup 11/24/2002 7:44PM: How many rectangles are on a chessboard?

You are a contestant on the Monty Hall game show. Three closed doors are shown before you. Behind one of these doors is a car; behind the other two are goats. The contestant does not know where the car is, but Monty Hall does. The contestant picks a door and Monty opens one of the remaining doors, one he knows doesn't hide the car. If the contestant has already chosen the correct door, Monty is equally likely to open either of the two remaining doors. After Monty has shown a goat behind the door that he opens, the contestant is always given the option to switch doors. What is the probability of winning the car if she stays with her first choice? What if she decides to switch?

MONTY HALL SHOW

Hint: Like many other problems on this site, the first answer that comes to mind tends to be wrong. Try enumerating the possible outcomes in a tree-like structure, recording the probabilities of each event along the way.

Note: This riddle was popularized by Marilyn vos Savant, current holder of the world's highest IQ. She introduced it in a magazine puzzle column, and was subsequently bombarded by flame mail accusing her of having the wrong solution, even though she was right. Even statistics professors were fooled! Today, this riddle is mentioned in almost every probability class.

You are in an empty room and you have a transparent glass of water. The glass is a right cylinder, and it looks like it's half full, but you're not sure. How can you accurately figure out whether the glass is half full, more than half full, or less than half full? You have no rulers or writing utensils. Hint 1: To help you get started if you're stuck, here's a solution that's not good enough. Holding the cup upright, use the palm of your left hand to cover the cup's opening. Now make a pinching gesture with the index finger and thumb of your right hand. Put the thumb at the base of the cup, and the index finger adjacent to the water level, thereby gauging the height of the water surface from the base of the cup. Now freeze the distance between those two fingers. Flip the cup upside down with your left hand; no water falls out since you've sealed the opening with

file:///C|/Documents%20and%20Settings/Administrator/M...20Documents/My%20Notepad/Jayesh%20placment/easy.shtml (7 of 28)8/7/2008 3:24:41 PM

[ wu :: riddles(easy) ]

GLASS HALF FULL

your left palm. Now put your frozen right hand against the cup, and see if the inverted water level is next to your index finger. If so, the cup is exactly half full. This seems like a good solution, but it's actually slightly inaccurate, because the palm of your hand is not a perfectly flat surface. Also, you'll most likely lose some water when you flip the cup upside down. We want a really accurate method. Hint 2: Utilize the geometry of the cup. That's really important. Hint 3: Don't drink the water. ^_^

Note: My friend David Lau found this riddle in a book designed for little kids. Hopefully you can solve it :)

You have a 6-foot long chain that is suspended at its ends, tacked to a wall. The tacks are parallel to the floor. Due to gravity, the middle part of the chain hangs down a little bit, forming a hump; the length of this hump in the vertical direction

HANGING CHAIN

is 3 feet. Find the distance in between the tacks. Note: asked at m$ interview.

Willywutang would like to have safe sex with three women, any of whom may be

TWO CONDOMS, THREE WOMEN

carrying an STD. Given two condoms, how can he do so, while ensuring that no STD is passed from one woman (or possibly himself) to another (or to himself)?

PRIME PAIRS M

Pairs of primes separated by a single number are called prime pairs. Examples are 17 and 19. Prove that the number between a prime pair is always divisible by 6 (assuming both numbers in the pair are greater than 6).

The second triangle is formed by rearranging pieces used to create the first. Yet there is a strange gap in the second triangle. Has area vanished? Is the conservation of matter bogus? Explain this madness.

MYSTERIOUS TRIANGLE AREA


file:///C|/Documents%20and%20Settings/Administrator/M...20Documents/My%20Notepad/Jayesh%20placment/easy.shtml (8 of 28)8/7/2008 3:24:41 PM

[ wu :: riddles(easy) ]

Hint: "Believe nothing, no matter where you read it, or who said it, no matter if I have said it, unless it agrees with your own reason and your common sense." Buddha

5 3 7 8 9 2 6 4 1 x

MYSTERY MATRIX I

8 0 2 0 4 What is x?

haha

What gets wetter the more it dries?

Place 8 queens on a chess board in such a way that they cannot capture each

8 QUEENS C

other. Note: asked at m$ interview.

A dragon and knight live on an island. This island has seven poisoned wells, numbered 1 to 7. If you drink from a well, you can only save yourself by drinking from a higher numbered well. Well 7 is located at the top of a high mountain, so only the dragon can reach it. One day they decide that the island isn't big enough for the two of them, and they have a duel. Each of them brings a glass of water to the duel, they exchange glasses, and drink. After the duel, the knight lives and the dragon dies. Why did the knight live? Why did the dragon die?

KNIGHT VS. DRAGON

file:///C|/Documents%20and%20Settings/Administrator/M...20Documents/My%20Notepad/Jayesh%20placment/easy.shtml (9 of 28)8/7/2008 3:24:41 PM

[ wu :: riddles(easy) ]

Note: From a Trilogy interview.

You and your arch rival are competing for the same girl. After years of battling, you both decide to settle it by tossing a coin. Your rival produces a coin, but you don't happen to have one on you. You are certain that the coin your rival has produced is loaded, ie. it will come up with heads more than 50% of the time on average. How do you arrange a fair contest, based purely on chance and not skill, by

COIN UNBIASING

flipping this coin? Variation: (COIN BIASING) You and your rival are competing for the same girl, and decide to settle it with a coin toss. Your rival has known the girl longer than you have, so you agree that it is fair for him to have a chance of winning equal to P, where P > 0.5. However, you only have a fair coin. How can you conduct this contest such that the biased probability is manifested? What is the average number of coin flips needed to determine a winner?

I am greater than God, and more evil than the devil. Poor people have me. Rich

WHO AM I? (1)

people want me. And if you eat me, you'll die. What am I?

You are a landscape specialist, and have been asked to design a garden for a

PLACING TREES

math professor. He wants four trees that are all equidistant from each other. How do you place the trees?

A boat of mass M1 is floating in a lake of water. The volume of the lake is V. The water surface is initially at height h, as measured relative to the lake's floor. There is an anchor of mass M2 sitting on the boat's deck. A person standing on deck picks up the anchor and throws it overboard. The anchor then sinks to the bottom of the lake, and the water surface height becomes h'. Which of the following qualitiative relationships is correct? What assumptions are

ANCHOR

you making about the values of M1, M2, h, and V? h' < h h' = h h' > h

Note: From the US Navy's nuclear power program interview for naval officers!

file:///C|/Documents%20and%20Settings/Administrator/...0Documents/My%20Notepad/Jayesh%20placment/easy.shtml (10 of 28)8/7/2008 3:24:41 PM

[ wu :: riddles(easy) ]

NEW DOOR SCRAMBLE

Can you rearrange the letters of new door to make one word?

You have two cylindrical rods of iron, identical in size and shape. One is a

MAGNET DETECTION

permanent magnet. The other is just non-magnetized iron -- attractable by magnets, but not permanently magnetic itself. Without any instrument, how can you determine which is which?

A cube is to be cut into 27 smaller cubes (just like a Rubik's Cube). It is clear that this can be done with 6 cuts to the original cube (2 in the x, 2 in the y, 2 in the z). Now, assuming that you can arrange the pieces however you like before doing a cut, what is the minimum number of cuts required to obtain the 27 smaller cubes? Prove your answer.

27 CUBES

ICE CREAM URBAN RIOTS

Scientific studies have discovered a direct, positive correlation between eating ice cream and the occurrence of massive urban riots. Why?

What is the least number of links you can cut in a chain of 21 links to be able to

CHAIN LINK 21

give someone all possible number of links up to 21?

A guy is sitting in some foreign country in death row awaiting his execution the next day. The executioner decides to grant him one last favor; he'll give him a choice in the execution method. The prisoner is therefore allowed to make one last statement. If this statement is true, he'll be hanged the next day. If however his statement is false he will be beheaded the next day. What should the prisoner say?

HANGING VS. BEHEADING

You have 9 dots arranged like a rectangle: . . . . . . . . .

9 DOTS
Without lifting your pen, draw four lines that cross all 9 dots. Good Hint: This is a very famous problem. It was actually responsible for the cliche: "Think outside the box."

file:///C|/Documents%20and%20Settings/Administrator/...0Documents/My%20Notepad/Jayesh%20placment/easy.shtml (11 of 28)8/7/2008 3:24:41 PM

[ wu :: riddles(easy) ]

222

Use the homophones "to", "too", and "two" in one question.

Do you agree with the following inductive proof? Clearly explain why or why not. Theorem: All horses are the same color. Base Case: 1 horse. Clearly with just 1 horse, all horses have the same color.

ALL HORSES SAME COLOR

Inductive Step: If it is true for any group of N horses that all have the same color, then it is true for any group of N+1 horses. Given any set of N+1 horses, if you exclude a random horse, you get a set of N horses. By the inductive step these N horses all have the same color. But by excluding any other horse in the pack of N +1 horses, you can conclude that the last N horses also have the same color. Therefore all N+1 horses have the same color. QED.

You have a car with a very flat roof, on a level road. There's a helium balloon in the car, barely scraping the roof - any slight force will move it. You start the car

BALLOON IN CAR

and accelerate forward very fast. Does the balloon move with respect to the car? If so, how? (This does not depend on wind from open windows or anything tricky.)

A Sheriff has captured a gang of ten desperados. His jail has only nine cells, and he cannot put more than one man into any one cell. What should he do? He tries taking the first two men and putting them into the first cell. The third deperado is put into the second cell, the fourth into the third, and the fifth into the fourth. The sixth, seventh, eigth and ninth men go into cells five, six, seven and eight respectively. Then then goes back to the first cell, where he originally put two men, and move the last man from there into cell nine. Has he solved the problem? Why or why not?

DESPERADOS IN JAIL

A jar contains one hundred marbles, each of which may be white or black. You pull out 100 marbles with replacement, and they are all white. What is the probability that all one hundred marbles are white?

MARBLE JARS II
Note: "With replacement" means you take out a random marble, look at its color, then put that marble back. Then repeat.

file:///C|/Documents%20and%20Settings/Administrator/...0Documents/My%20Notepad/Jayesh%20placment/easy.shtml (12 of 28)8/7/2008 3:24:41 PM

[ wu :: riddles(easy) ]

Im going to ask you if there are more than 6.02 x 10^23 stars in the universe.

HOW MANY STARS IN THE UNIVERSE?

Write the answer on a piece of paper. Make sure that everyone will agree you have written the correct answer on the paper.

In a certain town lived a miller, his daughter, and the evil mayor. The miller was in debt to the mayor, and the mayor had his eye on the millers daughter. The mayor made a proposition: he would place a black stone and a white stone in a bag, and the millers daughter would pick one out in front of the whole village. If she drew the white stone, the mayor would forgive the millers debt. If she drew the black stone, the mayor would marry the millers daughter and take the mill. The miller had no choice but to agree. The millers daughter has no reason to trust the mayor, and believes that he will place two black stones in the bag. How can she get out of marrying the mayor and save the mill?

EVIL MAYOR

The overly feminist rulers of a country decide that there are too many baby boys being born. The rulers decide to enforce a new law concerning child birth on their overly prosperous subjects. Each family is permitted to have as many children as they want, provided that they only produce baby girls. Once a baby boy enters the family, the family is no longer permitted to have children. Assuming each law abiding family wants to have as many children as possible, what will happen to the ratio of boys to girls, and why? the ratio of boys to girls will go up. the ratio of boys to girls will stay the same. the ratio of boys to girls will go down.

FEMINIST RULE

Click here to listen to the problem statement. Now highlight the area below with your mouse to see the partial sequence: WITNLIT_

LETTER SEQUENCE

You're looking through a hole, at the corner of a regular, normal die. The below image shows all that you can see. Can you identify AT LEAST ONE of the sides visible through the hole?

DICE CORNER

file:///C|/Documents%20and%20Settings/Administrator/...0Documents/My%20Notepad/Jayesh%20placment/easy.shtml (13 of 28)8/7/2008 3:24:41 PM

[ wu :: riddles(easy) ]

A man is 3/8's of the way across a train bridge, when he hears the whistle of an approaching train behind him. It turns out that he can run in either direction and just barely make it off the bridge before getting hit. If he is running at 15 mph, how fast is the train traveling? Assume the train travels at a constant speed, despite seeing you on the tracks. Note: From a 7th grade pre-algebra book.

TRAIN BRIDGE

What goes in the blank? _ T S T S T E T E F N F N F T F T

MYSTERY MATRIX II

S E S

Hint: it's not 'E'

A rich old man has died. After his death, his children are surprised to learn that he has left all of his money to his oldest son Jeremiah, who loved him dearly, and ignored his other children, who hated him. So, the funeral is a day or two later, and the other sons and daughters have decided to kill Jeremiah and take his inheritance. Since his father's death, Jeremiah has taken to drinking, and they know that, at the wake, he's going to be gulping down the liquor like it was nectar of the gods. So they decide to poison the drinks. One of the other sons, Wallace, tends bar, and gets the poison all ready. So Jeremiah comes up, crying and depressed, and orders a scotch on the rocks.

POISON DRINKS

Wallace serves him one, and he chugs it down in two seconds. "Give me another." Wallace gives him a second glass of scotch, which he also drinks in a matter of moments. The other siblings are puzzled...the poison is fast-acting; Jeremiah should be convulsing on the floor and retching his guts out. Finally, fifteen minutes later, a rather inebriated and very much alive Jeremiah orders one last

file:///C|/Documents%20and%20Settings/Administrator/...0Documents/My%20Notepad/Jayesh%20placment/easy.shtml (14 of 28)8/7/2008 3:24:41 PM

[ wu :: riddles(easy) ]

glass of scotch, but as Wallace hands it to him, he changes his mind and leaves, sobbing. The other siblings come over to Wallace, and wonder what's going on. They talk about what could have gone wrong for a few minutes, and figure the poison's harmless. So Wallace sips the drink he poured for Jeremiah, and is pronounced DOA thirty minutes later. Why did Jeremiah live? (He had no immunity to the poison, he didn't know it was coming, and the poison was obviously deadly.)

A 12 by 25 by 36 inch box is lying on the floor on one of its 25 by 36 inch faces. An ant, located at one of the bottom corners of the box, must crawl along the outside of the box to reach the opposite bottom corner. It can walk on any of the box faces except for the bottom face, which is in flush contact with the floor. What is the length of the shortest such path?

ANT ON A BOX

Sam and Max run a 100 meter race. Sam wins by five yards. To make it sporting, he starts 5 yards beind the original start line in the second race. Assuming both

100 M RACE

runners run at the same speed, who wins the second race? The challenge is to solve this problem without doing any algebra.

Bill has two girlfriends, Hillary and Monica. Monica lives in the East of a city, and Hillary lives in the West of the same city, as shown in the figure below. Once every morning at a random time, Bill arrives at the train station at the center of the city. A train leaves for the East every 10 minutes, and a train leaves for the West every 10 minutes Bill chooses whichever train arrives first. On average, could Bill end up with one girl more often than the other? If so, how many times more often? Why?

TWO GIRLFRIEND PARADOX

file:///C|/Documents%20and%20Settings/Administrator/...0Documents/My%20Notepad/Jayesh%20placment/easy.shtml (15 of 28)8/7/2008 3:24:41 PM

[ wu :: riddles(easy) ]

Mathematicians normally disparage ambiguity and sensitize themselves to its symptoms, so as to detect and correct it, more than do many other intellectuals. For example, intelligence tests used by American MENSA, a self-styled American High I.Q. Society, are notorious for unintended ambiguities that elicit incorrect responses from more imaginative and intelligent test takers, thus thwarting the tests ostensible purposes. It's rather ironic. The following questions, framed by a MENSA psychologist, came from a box of Raisin Bran &copy. For each question devise as many answers as you can, all at least as valid as the one answer the psychologist deemed correct.

1. Which of the following five words doesn't belong with the others, and why?

pail

skillet

knife

suitcase

card

2. One of the figures below lacks a characteristic common to the other figures. Which

SMARTER THAN MENSA

one, and why?

3. One of the figures below lacks a characteristic common to the other figures. Which one, and why?

English grammar used to be taught as an analytical subject, but today such rigorous treatment is rarely seen in the States. Consequently, most modernized Americans are unable to discern the differences between the following four sentences: Only birds read poetry. Birds only read poetry. Birds only read poetry. (two different interpretations for this sentence exist)

GRAMMAR

file:///C|/Documents%20and%20Settings/Administrator/...0Documents/My%20Notepad/Jayesh%20placment/easy.shtml (16 of 28)8/7/2008 3:24:41 PM

[ wu :: riddles(easy) ]

DISTINCTIONS

Birds read only poetry.

For each of these sentences, write a sentence or two showing that you appreciate the distinctions. Bonus Question: Translate each of the above sentences into formalized logical expressions, using boolean logic symbols and quantifiers (e.g. and, or, not, implies, for all, there exists, etc).

E, O, E, R, E, ?,

MYSTERY MATRIX III

N, ?, E, N What goes in the question marks?

A confused bank teller transposed the dollars and cents when he cashed a check

CONFUSED BANK TELLER

for Ms Smith, giving her dollars instead of cents and cents instead of dollars. After buying a newspaper for 50 cents, Ms Smith noticed that she had left exactly three times as much as the original check. What was the amount of the check?

Two ladders are placed cross-wise in an alley to form a lopsided X-shape. Both walls of the alley are perpendicular to the ground. The top of the longer ladder touches the alley wall 5 feet higher than the top of the shorter ladder touches the opposite wall, which in turn is 4 feet higher than the intersection of the two ladders. How high above the ground is that intersection?

TWO LADDERS

The NSA has a large number of spy satellites in geosynchronous orbit; if I told you the exact number, I'd have to kill you. These satellites communicate continuously by microwaves with stations on the Earth, and with each other --

SPY SATELLITES

except when the Earths bulk interrupts the line-of-sight path that microwaves need. Prove that at all times, at least two satellites are each in uninterrupted communication with the same number of satellites.

file:///C|/Documents%20and%20Settings/Administrator/...0Documents/My%20Notepad/Jayesh%20placment/easy.shtml (17 of 28)8/7/2008 3:24:41 PM

[ wu :: riddles(easy) ]

Violins produced on the island of Grxcd have become collectors items since it sank into the sea two centuries ago. All the islands violins were produced by Bropcs or one of his sons, or by Czwyz or one of his sons. Every violin was labelled ostensibly to reveal its maker but, although Bropcs and his sons always labelled their violins truthfully, Czwyz and his sons always labelled their violins with falsehoods. Both families playfully interfered with collectors attempts to establish provenances for their violins. For example, collectors figured out that a violin labelled This violin was not made by any son of Bropcs. was made by Bropcs Sr.; can you see why? The most desirable violins are so labelled that a connoisseur can tell that it must have been made by one of the fathers, either Bropcs Sr. or Czwyz Sr., but cannot tell which. How might such a violin be labelled?

COLLECTOR VIOLINS

In this variation on the game of Poker, two people play as follows: Player 1 takes any 5 cards of his choice from the deck of 52 cards. Then player 2 does the same out of the remaining 47. Then player 1 may choose to discard any of his cards and replace them from the remaining 42. Then player 2 may discard any of his cards and replace them, but he may not take player 1's discards. ALL of the transactions with the deck are public knowledge, unlike the real game of Poker. After this process, the winner is the one who has the better poker hand. For the benefit of those who have not played poker, these are the highest ranking hands, in decreasing order of value: 1. Royal Flush: the A K Q J 10 of the same suit. 2. Straight Flush: any five consecutive of one suit. Highest card of the five is the tiebreaker. No one suit is more powerful than another. 3. Four of a kind: all four of one rank (i.e. four aces). A hand with 4 aces outranks 4 kings, etc. 4. Full house: a pair of one rank and 3-of-a-kind in another rank, i.e. Q Q 8 8 8. 5. Flush: Any 5 cards of the same suit that don't satisfy #2. Because of the clear advantage of player 1, the win is given to player 2 if the hands are equal in strength. Which player would you rather be? What strategy do you use?

OPEN POKER

You are given n coins of denominations 1, 0.5, 0.25, 0.1, 0.05 and 0.01 (6n coins

MAKING A DOLLAR

altogether). You are then asked to choose n out of these 6n coins that sum up to exactly 1. What is the smallest n for which this is impossible?

file:///C|/Documents%20and%20Settings/Administrator/...0Documents/My%20Notepad/Jayesh%20placment/easy.shtml (18 of 28)8/7/2008 3:24:41 PM

[ wu :: riddles(easy) ]

A certain UberPuzzler in a certain puzzle forum uses the signature "All signatures are false". What is the most that can be deduced from this statement alone (i.e. without any knowledge of other signatures)? ( with apologies to J.F. )

ALL SIGS ARE FALSE

Note: If you liked this recursive statement, you might also like "Why is there no correct answer to this question?" and "A man comes up to you and says 'I am lying.' Can you conclude anything?"

A stopped clock gives the exact time twice a day, while a normally running (but out of sync) clock will not be right more than once over a period of months. A

THE CLEVER GRANDFATHER

clever grandfather [as in grandfather clock] adjusted his clock to give the correct time at least twice a day, while running at the normal rate. Assuming he was not able to set it perfectly, how did he do it?

On a geography test you have to tell which of two German cities is greater in population for all possible pairs of the 80 largest cities of Germany. (And that's the only task on the test since it's already 5 pages long.) But you didn't study last night, and only even recognize half the cities, and don't even know how those are ordered relative to each other. Your friend on the other hand studied dutifully all

GEOGRAPHY TEST

night and recognizes all the cities and even knows how two cities are ranked relative to each other 60% of the time. A week later you get the test-result and you have a higher score than your friend. How come?

LOG MOTOR P
There's a perfectly cylindrical log mounted horizontally on frictionless pins at
file:///C|/Documents%20and%20Settings/Administrator/...0Documents/My%20Notepad/Jayesh%20placment/easy.shtml (19 of 28)8/7/2008 3:24:41 PM

[ wu :: riddles(easy) ]

each end. It is in a container, set up so that, looking down the length of the log, on one side is air and the other is water. There are walls to keep the air and water separated, and these walls meet the log lengthwise with frictionless seals. Given that a log floats in water, would the log start spinning? Why or why not? See diagram for cross-sectional view.

Note: Originally an interview question for a mechanical engineering position!

Consider a list of 2000 statements: 1) Exactly one statement on this list is false. 2) Exactly two statements on this list are false. 3) Exactly three statements on this list are false. . . . 2000) Exactly 2000 statements on this list are false. Which statements are true and which are false? What happens if you replace "exactly" with "at least"?

TRUE OR FALSE LIST

What happens if you replace "exactly" with "at most"? What happens in all three cases if you replace "false" with "true"?

Note: "The 'exactly . . . false' problem was posed by David L. Silverman for 1969 statements in the January, 1969 issue of the Journal of Recreational Mathematics. I got the problem from Martin Gardner's "Knotted Doughnuts and Other Mathematical Entertainments", where he discusses it and some of the variants above." - Paul Sinclair

There is a rich man living with two other people his butler and his maid. One day the rich man is sitting at his desk counting his money, preparing to deposit it at the bank. When he is done he goes to the bank, but when he arrives, he realizes he left a $100 dollar bill on the desk. So he quickly called the house and told the butler that he forgot the $100 bill on the desk, and he will come home now to pick it up. When he arrived home he asked the butler what he did with the bill, the butler said he put it under the green book on the desk. When the rich man looked under the book, it wasn't there, so he asked the maid if she saw it. She said she saw the $100 bill when she was dusting and put it between pages 67 and 68 of the green book. Right then and there the rich man called the police, and knew who stole it, even before the rich man checked inside the book to see if the $100 bill was there. How did he know?
file:///C|/Documents%20and%20Settings/Administrator/...0Documents/My%20Notepad/Jayesh%20placment/easy.shtml (20 of 28)8/7/2008 3:24:41 PM

MONEY IN BOOK

[ wu :: riddles(easy) ]

Note: Technically the book must have a certain property for this trick to work, but you would be hard pressed to find a book without this property.

CENTURY FINAL DAY

Assuming year 1 started on Sunday, it can be shown that only some days are possible as a century's final day. What are these days?

(Classic puzzle) A farmer returning home from the market must get across the river and return home with his three purchases, a dog, a chicken and a bag of rice. However, He must take them in his boat. He can't have more than one item with him on his boat at all times. He cannot leave the dog alone with the chicken

DOG, CHICKEN, RICE

because the dog will eat the chicken, and he cannot leave the chicken alone with the bag of grain because the chicken will eat the bag of grain. How does he get all three of his purchases back home safely? Note: (1/19/2003 12:59AM) Corrected a wording error.

There is a tree 20 feet high, with a circumference of 3 feet. A vine starts at the base of the tree and winds around the tree 7 times before reaching the top. How long is the vine? Hint: There is an easy way to solve this problem which only uses junior high

WINDING VINE LENGTH

school math!

Note 1: Apparently from Chinese texts over 2000 years old. Note 2: Treat the tree as a perfect cylinder.

You walk into a room in which there are three primates: a chimpanzee, an orangutan, and a gorilla. The chimpanzee is holding a banana in each hand, the

THREE PRIMATES

orangutan is holding a big stick, and the gorilla is holding nothing. Which primate in the room is the smartest?

HOW MANY LETTERS IN CORRECT ANSWER

How many letters does the correct answer to this puzzle contain?

file:///C|/Documents%20and%20Settings/Administrator/...0Documents/My%20Notepad/Jayesh%20placment/easy.shtml (21 of 28)8/7/2008 3:24:41 PM

[ wu :: riddles(easy) ]

A patient has fallen very ill and has been advised to take exactly one pill of medicine X and exactly one pill of medicine Y each day, lest he die from either illness or overdosage. These pills must be taken together. The patient has bottles of X pills and Y pills. He puts one of the X pills in his hand. Then while tilting the bottle of Y pills, two Y pills accidentally fall out. Now there are three pills in his hand. Because both types of pill look identical, he cannot tell which two pills are type Y and which is type X. Since the pills are extremely expensive, the patient does not wish to throw away the ones in his hand. How can he save the pills in his hand and still maintain a proper daily dosage?

TAKING PILLS

Given the following information, what is 10 + 10? 1+1=0; 2+2=0; 3+3=0; 4+4=2; 5+5=0; 6+6=2; 7+7=0; 8+8=4; 9+9=2; 10+10=?

JAPANESE KINDERGARTEN APPLICATION

Note: Supposedly this riddle is from an application to a Japanese kindergarten! Amusingly I didn't get the correct solution myself, so I guess my academic career would've been toasted real early if I lived in Japan. God Bless America! =D

A bunch of nifty match configuration problems from forum regular [BNC]. Starting with the following configuration: - | | | - | | | - -

MATCH PUZZLES
1. Remove two matches to get two squares -- one larger than the other. 2. Move 3 matched to get 3 identical squares. 3. Move 4 matched to get 3 identical squares. 4. Move 2 matches to get 7 (non-identical) squares. hint: you may place one match over another 5. Move 4 matches to get 10 (non-identical) squares. 6. Move 8 matches to get 6 identical squares.

file:///C|/Documents%20and%20Settings/Administrator/...0Documents/My%20Notepad/Jayesh%20placment/easy.shtml (22 of 28)8/7/2008 3:24:41 PM

[ wu :: riddles(easy) ]

2/4/2003 2:55AM Willywutang looked at the piece of wood before him. It is a 10 diameter round, flat piece he wanted to use as a wheel in his art lesson homework. Alas! While drilling the center hole (1 diameter) , he sneezed, and the hole is way off-center (although, luckily, still within the wheel the center of the hole is 3 away from the center of the wheel).

OFF-CENTER WHEEL

And then, revelation! Willy cuts the wheel into two parts, glued them again, and got a perfectly centered hole in the wheel. How did he do it?

Note: assume Willy has in his possession a magical cutter that cuts with zero width, and magic glue that can glue pieces with zero distance between them.

2/4/2003 2:55AM In each of the following three configurations of matches, move a single match to form valid equations. The = and + are composed of two matches each.

MATCH EQUATIONS
1. \/|+|=\/ 2. \/|+|=\/| 3. \/|+|=|||

2/4/2003 2:55AM How quickly can you find out what is so unusual about this paragraph? It looks so ordinary that you would think that nothing is wrong with it at all, and, in fact, nothing is. But it is unusual. Why? If you look at it, study it and think about it, you may find out, but I am not going to assist you in any way. You must do it without coaching. No doubt, if you work at it for long, it will dawn on you. Who knows? Go to work and try your skill. Par is about half an hour. So jump to it and try your skill at figuring it out. Good luck --don't blow your cool.

UNUSUAL PARAGRAPH

Hint 1: I thought long and hard about this. "RJ, " I said, "a solution is probably right in front of you. Just chill out and it will just pop into mind." I sat and thought and sat and thought but I'm still waiting. I'm probably not as smart as I think I am. - R. Jacobus Hint 2: RJ, don't quit. You'll find that paragraph's odd quality, but you must stay with it. My old typing instructor taught my class this quizzical brain-twisting oddity many moons ago, so I had a solution right away, but I know you can do it if you studiously put your mind to it. - Wolfgang

file:///C|/Documents%20and%20Settings/Administrator/...0Documents/My%20Notepad/Jayesh%20placment/easy.shtml (23 of 28)8/7/2008 3:24:41 PM

[ wu :: riddles(easy) ]

2/4/2003 2:55AM

TWO CONSECUTIVE HEADS

An unbiased coin is tossed n times. What is the probability that no two consecutive heads appear?

3/9/2003 4:08AM So, an eccentric entrepreneur by the name of Alphonse Null has sent out a press release about his new, mind-blowing hotel: The Hotel Infinity. Null informs the world that this hotel has an infinite number of rooms (specifically, an infinity equal to the cardinality of the integers). A quick tour puts skeptics' claims to rest; as far as anyone can tell, this hotel has infinite rooms. The consequences are mind-boggling, and Null sets up a press conference to answer questions... "So, Mr. Null, how will patrons get to their room, if their room number has, say, more digits than protons in the universe?" "The elevators have an ingenious formula device instead of buttons... simply input the formula for your room number, with Ackermann numbers or somesuch... your room formula can be picked up at the front desk. There's not even any need to know what the formula means!" "How do you produce the power and water for this hotel?" "I have infinite generators and wells, of course. This IS an infinite hotel, you know! *chuckle*" "What about costs? How much will it cost to stay here?"

HOTEL INFINITY

"That's the beauty of it! Since there are as many positive even integers as there are integers, I can change the same price to only every other room and still make the same profit! I could charge only every millionth room... each guest has a onein-a-million chance of not getting a free room, and I still get paid the same! I love the properties of infinite sets, especially when it comes to profit!" "But, Mr. Null... I think you've made a severe mistake in your assumptions regarding profit..." "Oh?" The reporter then mentioned something which made Mr. Null's face turn white. "Oh... oh goodness... THIS PRESS CONFERENCE IS OVER!" Then he ran out. Assuming that everything Null said about the hotel is true: it really is infinite; it really is easy to get to your room; it really can generate infinite power for the

file:///C|/Documents%20and%20Settings/Administrator/...0Documents/My%20Notepad/Jayesh%20placment/easy.shtml (24 of 28)8/7/2008 3:24:41 PM

[ wu :: riddles(easy) ]

guests; the cardinality of the set of multiples of a million, is the same as the cardinality of the integers... So with what simple assumption did Mr. Null go wrong? Note: "(There's one assumption I'm looking for, although any other assumption which would work is fine too.)" - Jeremiah Smith, writer of this puzzle

3/9/2003 4:11AM It is known that a quadratic equation has either 0, 1, or 2 unique real solutions. Well, look at this equation:

TOO MANY SOLUTIONS

Without loss of generality, assume a < b < c. Now note that x=a, x=b, and x=c are all unique solutions! How can this equation have 3 solutions?!

4/7/2003 12:17AM The many incarnations of these simple online "mind-reading" magic tricks have stupefied many a surfer. Hopefully you will not be duped too? Mind-Reading Card Selection: http://sprott.physics.wisc.edu/pickover/esp. html
q

ESP PUZZLES

Mind-Reading Symbol Selection: http://mr-31238.mr.valuehost.co.uk/ assets/Flash/psychic.swf

Note: Try to rigorously explain what makes the second trick work.

4/7/2003 12:17AM You are given 2 identical looking spheres. They have the same mass and have the same diameter. Physically, they look the same, and have the same surface texture. (ie you can't visually pick them apart) They are both hard, thus they won't bounce and they won't have any 'give'. They both have perfectly smooth surface. One is made of less dense material and is soild and uniform through out. The

TWO IDENTICAL SPHERES

other is made of higher density material, but since having the same mass and volume as the other, it is hollow at its centre (assume a spherical cavity with the centre of cavity and centre of the whole sphere at the same point).

file:///C|/Documents%20and%20Settings/Administrator/...0Documents/My%20Notepad/Jayesh%20placment/easy.shtml (25 of 28)8/7/2008 3:24:41 PM

[ wu :: riddles(easy) ]

With a minimum of instruments, how can you determine which one is hollow and which one is solid?

Note: (4/7/2003 12:36AM) Many of my friends here at UC Berkeley have been asked this question at tech interviews recently (Microsoft, Amazon.com, etc.)

4/7/2003 12:17AM The boiling point of olive oil is higher than the melting point of tin. If Italian skillets are made of tinned copper, how can they be used to fry food in olive oil?

FRYING FOOD

Note: Originally posed to Enrico Fermi by one of his students.

4/7/2003 12:17AM

21 FACTORIAL M

21!=510909x21y1709440000 Without calculating 21!, what are the digits marked x and y?

5/11/2003 10:47PM Consider a bicycle as shown in the picture below. It is perfectly normal except for a piece of string caught in the rear wheel. If we pull the string in the direction P, will the bicycle move forward, move backward, or 'stay put'? Assume that the wheel does not slip on the ground.

BICYCLE TUGGING

5/11/2003 10:59PM

SUM OF REAL NUMBERS

The sum of N real numbers (not necessarily unique) is 20. The sum of the 3 smallest of these numbers is 5. The sum of the 3 largest is 7. What is N?

file:///C|/Documents%20and%20Settings/Administrator/...0Documents/My%20Notepad/Jayesh%20placment/easy.shtml (26 of 28)8/7/2008 3:24:41 PM

[ wu :: riddles(easy) ]

5/11/2003 10:59PM Two people come to a river. There is a boat, however it can carry one person only. How can they each get to the other side of the river using the boat?

RIVER CROSSING
Note: The problem is too open-ended to have only one solution, but think of the most elegant possible scenario.

Contributor: William Wu

5/11/2003 10:59PM

TWIDDLING BOLTS

The bolts shown above have regular helical grooves. If you circle the bolts around each other in the directions indicated, in the way you would twiddle your thumbs, will the bolt heads: move away from each other, move toward each other, or stay the same distance apart?

Explain your reasoning. Note that you do not rotate either bolt around its own axis, and you always keep the bolts closely in contact with one another.

Contributor: Nootch

9:00 PM 8/19/2004

"What's the Dominant Fifth?" asked Dr. Dingo, as his daughter Cicely came in from school. Cicely blushed. "Just a secret society," she said. "I'm one of the vice-presidents." "And you're meeting tonight; is that right?" "How on earth did you know?" ask Cicely. "You left this lying about. That's no way to keep secrets, my girl." He handed

DOMINANT FIFTH

Cicely this paper: Dominant Fifth

file:///C|/Documents%20and%20Settings/Administrator/...0Documents/My%20Notepad/Jayesh%20placment/easy.shtml (27 of 28)8/7/2008 3:24:41 PM

[ wu :: riddles(easy) ]

REASM NCNVE OTMLE SEHST TAOEI "How did you manage to read it?" asked Cicely. "The code is known to only about eight of us." "Change it," said Dingo. "Any fool can read that." Where and When is the next meeting scheduled?

Contributor: William Wu

9:00 PM 8/19/2004

You are given a finite sequence of n real values (a1,...,an). Prove that for any

CUTE PERMUTATION INEQUALITY

permutation

of {1,...,n}, ai a <= ai2

i=1 to n

(i)

i=1 to n

At least one two-liner quickie proof exists.

Thanks to Contributors

David Lau, Siddhartha Doshi, Jiong Shen, Carl Wang, Phoebus Chen, Alan Liu, Hansen Bow, Ernest Zhu, Elaine Lo, Yosen Lin, Don Barkauskas, Katherine Chan, Jasvir Nagra, Tau Beta Pi (TBP), Eta Kappa Nu (HKN), Danny Dulai, His Grace The Duke Of Ankh-Morpork Commander Sir Samuel Vimes, Michael Snyder, Dipan K. Ghosh, Eric Cole, Louis Wainwright, Ben Bardill, Patrick Dreker, Autumn Looijen, Stephen Champailler, Christopher Kings-Lynne, Bart Samwel, Kannan Ramchandran, Nick Yee, Steve Plimpton, Levsen Hendrik, Remco Hartog, [I.M._Smarter_Enyu], Philip Mock, Michael Chang, Jon Meilstrup, Ryan Russell, Matt Young, Jonathan Haas, Geoff Canyon, Peter Surda, Cory Ondrejka, Satish Rao, [gcooper], Ted Powell, Brave Sir Robin, Eric Cole, J. A. H. Hunter, Sean R. Owen, Andrew Glenn, Bruce Preston, Peter Ratiu, Michael Mendelsohn, Rob Mahurin, James Fingas, Bryan Organ, Jeroen Rutten, Stephen Montgomery-Smith, Marko Lukat, Eric Yeh, Nick Hobson, Mike Lawther, [anshil], Richard Feynman, Douglas Hofstaeder, Dacher Keltner, David Mace, [SAS], Matthew Schultheis, John Leen, Andrew Ooi, Folkert Hindriks, Steve Ragle, Daniel Filner, Karl Barrus, Misha Kruk, Keith Lloyd, Dave Minott, Jette Randlov, Eric Winger, Nathan Hellweg, Tom VanCourt, Chris Seaton, Mitchell Morris, Michael Styer, Zameer Andani, Jonathan Blow, Jeff Thompson, Jonathon Duerig, Dan Hanson, Gabriel Sechan, Tom Saxton, [HunterWare], [alsee], James Antill, Tom Barringer, Bart Massey, David Krikorian, Eric Sharkey, [tudorb], Kevin Day, Milan Ramaiya, Robert Merkel, James Jones, Haim Bitner, Adam Barth, Oscar Lazzarino, Damien Fisher, [DrkShadow], Erik Blankendaal, Eric Smith, James Demmel, Jonathan Shewchuk, Alex Harris, Michael Kelley, [Mr._Martingale], Kaisen Lin, Hakan Yilmaz, Freddy Mercury, Justin Rising, Marko Lukat, William Kahan, Jeremy Randolph, Michael Sinatra, David Messerschmitt, Patrick Masterson, Frederik Bonte, Randy Williams, Pietro K.C., Brett Danaher, Derek Abbott, Ralph Boleslavsky, Rui del-Negro, [college math journal], [amer. math monthly], Spyros Potamianos, Gary Hsieh, [rec.puzzles], Steven Rudich, Matt Lahut, Richard Reti, Paul Sinclair, Tim Mann, [ucb engineering news], Luke Percival, Anwis Das, Mike White, Louise Poon, Jeffrey Wilhelm, Anthony Cammon, [BNC], A.Frieze & D.Sleator, [SWF], Ted Stevens, Frank Wang, Danny P, Patrick Sesulka, [towr], Chi Sum Cheung, Ranjit Jhala, Jacob Scott, David McKay, Eamon Warnock (THUDandBLUNDER), Kozo Morimoto, Abhijit Joshi, Devesh Parekh, Amnon Melzer, Mary Lou, Leonid Brouhkis, Allistair Sinclair, Mark Newheiser, Joc Koelman, Paul Jung, Aryabhatta, Thomas Cover, Barukh, Nootch, Eigenray

William Wu 2002-04: home | intro | easy | medium | hard | microsoft | cs | putnam | cigs | faq | pros | cons | laff | forum | credits

file:///C|/Documents%20and%20Settings/Administrator/...0Documents/My%20Notepad/Jayesh%20placment/easy.shtml (28 of 28)8/7/2008 3:24:41 PM

[ wu :: riddles(medium) ]

Stuck? Have compliments or criticisms? Want to test-drive a new riddle of your own? Who are we, and what do we do? Visit the fantabulous riddle forum! Thousands of posts by really clever people.

RECENT ADDITIONS
Check out latest puzzles by perusing the forum and the 10 most recent posts. Latest additions to cover site can be seen by clicking here.

relatively medium
how many places are there on the earth that one could walk one mile south, then one mile west, then one mile north and end up in the same spot? to be precise, let's assume the earth is a solid smooth sphere, so oceans and mountains and other such things do not exist. you can start at any point on the sphere. also, the rotation of the earth has nothing to do with the solution; you can assume you're walking on a static sphere if that makes the problem less complicated to you. Hint 1: think you've figured it out? do you know that there's more than one? in fact, there are more than two. also note that walking north from the north pole (or south from the south pole) is illogical and therefore does not enter into the problem. all normal assumptions about directions will be used. Hint 2: christopher columbus.

GLOBE TRAVERSAL

file:///C|/Documents%20and%20Settings/Administrator/...ocuments/My%20Notepad/Jayesh%20placment/medium.shtml (2 of 32)8/7/2008 3:24:44 PM

[ wu :: riddles(medium) ]

You want to send a valuable object to a friend securely. You have a box which can be fitted with multiple locks, and you have several locks and their corresponding keys. However, your friend does not have any keys to your locks, and if you send a key in an unlocked box, the key could be copied en route. How can you send the object securely? Alternative, more precise phrasing: Andy and Grant are staying in different rooms in the same hotel. Andy needs to give a gold pendant to Grant, but spies are trying to assassinate Andy and

GHETTO ENCRYPTION I

Grant so neither of them can leave their room. The only way they can transfer objects is by using the bellhops. Both Andy and Grant have a safe with a large clasp that can be secured with a padlock. Both Andy and Grant have a padlock and a corresponding key. (So 1 gold pendant, 2 safes, 2 padlocks, and 2 keys.) But the bellhops are thieves. Anything that is not padlocked in the safe will be stolen by the bellhops - including any unlocked padlocks, the keys or the pendant. How can Andy transfer the gold pendant to Grant without it being stolen? (where both sides have encryption capability, and where unsecured items are taken away rather than just copied?)

Three coworkers would like to know their average salary. However, they are self-conscious and don't want to tell each other their own

GHETTO ENCRYPTION II

salaries, for fear of either being ridiculed or getting their houses robbed. How can they find their average salary, without disclosing their own salaries?

file:///C|/Documents%20and%20Settings/Administrator/...ocuments/My%20Notepad/Jayesh%20placment/medium.shtml (3 of 32)8/7/2008 3:24:44 PM

[ wu :: riddles(medium) ]

arrange the numbers 1 to 8 in the grid below such that adjacent numbers are not in adjacent boxes (horizontally, vertically, or diagonally).

ADJACENCY GRID

6 2

4 7 8

3 5

the arrangement above, for example, is wrong because 3 and 4, 4 and 5, 6 and 7, and 7 and 8 are adjacent.

you die and the devil says he'll let you go to heaven if you beat him in a game. the devil sits you down at a round table. he gives himself and you a huge pile of quarters. he says "ok, we'll take turns putting quarters down, no overlapping allowed, and the quarters must rest on the table surface. the first guy who can't put a quarter down loses." you guys are about to start playing, and the devil says that he'll go first. however, at this point you immediately interject, and ask if you can go first instead. you make this interjection because you are very smart, and you know that if you go first, you can guarantee victory. explain how you can guarantee victory.

FAUSTIAN ROUND TABLE COIN GAME

using 31 dominoes, where one domino covers exactly two squares, can you cover all the empty squares on this chessboard (which has only 62 spaces, since two opposite corner squares are removed). if so, how? if not, why? prove your claim.

DOMINOES ON A CHESSBOARD

file:///C|/Documents%20and%20Settings/Administrator/...ocuments/My%20Notepad/Jayesh%20placment/medium.shtml (4 of 32)8/7/2008 3:24:44 PM

[ wu :: riddles(medium) ]

Why are manholes round?

MANHOLES

Note: This is a famous Microsoft question. Yet amusingly, the Microsoft campus uses square manholes.

SQUARE DIVISION

draw a square. divide it into four identical squares. remove the bottom left hand square. now divide the resulting shape into four identical shapes.

EQUILATERAL TRIANGLE DIVISION

draw an equilateral triangle (all sides same length). divide it into four identical shapes. remove the bottom left hand shape. now divide the resulting shape into four identical shapes.

You are in a room with three light switches, each of which controls one of three light bulbs in the next room. Your task is to determine which switch controls which bulb. All lights are initially off, and you can't see into one room from the other. You are allowed only one chance to enter the room with the light bulbs. How can you determine which lightswitch goes with which light bulb?

LIGHT BULBS AND SWITCHES

Add punctuation to the following phrase to make something gramatically and logically coherent:

PUNCTUATION I
is is not not not is not is is is is not is not is it not

file:///C|/Documents%20and%20Settings/Administrator/...ocuments/My%20Notepad/Jayesh%20placment/medium.shtml (5 of 32)8/7/2008 3:24:44 PM

[ wu :: riddles(medium) ]

You have a 5x5 piece of paper. Two diagonally opposite corners of this paper are truncated as shown in the diagram below. You also have scissors. Show how to cut up the 5x5 paper into two pieces, so that the two pieces can then be interlocked to form a 6x4 rectangle.

PAPER CUTTING

In the city of Funkytown, the following facts are true: No two inhabitants have exactly the same number of hairs. No inhabitant has exactly 483,207 hairs. There are more inhabitants than there are hairs on the head of any one inhabitant. What is the largest possible number of inhabitants of Funkytown? Note: I recently (11/24/2002 8:00PM) read this puzzle in a book by Henry Ernest Dudeney, England's greatest puzzle creator. So writing credits to him.

POPULATION OF FUNKYTOWN

You have two thermoses. The first contains a liter of milk, the second contains a liter of pure chocolate syrup. You pour one cup

CHOCOLATE MILK

of milk out from the first thermos to the second one. Then, after mixing that, you take one cup of the mixture from the second thermos, and pour it back into the first thermos. After completing these two operations, which thermos is more pure?

file:///C|/Documents%20and%20Settings/Administrator/...ocuments/My%20Notepad/Jayesh%20placment/medium.shtml (6 of 32)8/7/2008 3:24:44 PM

[ wu :: riddles(medium) ]

There is an island of monks where everyone has either brown eyes or red eyes. Monks who have red eyes are cursed, and are supposed to commit suicide at midnight. However, no one ever talks about what color eyes they have, because the monks have a vow of silence. Also, there are no reflective surfaces on the whole island. Thus, no one knows their own eye color; they can only see the eye colors of other people, and not say anything about them. Life goes on, with brown-eyed monks and red-eyed monks living happily together in peace, and no one ever committing suicide. Then one day a tourist visits the island monastery, and, not knowing that he's not supposed to talk about eyes, he states the observation "At least one of you has red eyes." Having acquired this new information, something dramatic happens among the monks. What happens?

BROWN EYES AND RED EYES

Hint: First consider the case where there are only a few monks on the island, some with brown and some with red. Work through the logic and find out what happens over time. Then generalize for the case of M monks on the island, N of which have red eyes.

Update 10/15/2002 12:14AM: What happens if we change the tourist's statement to each of the following? 1. "There are 10 Brown Eyed Monks" 2. "There are at lesat two Red Eyed Monks" 3. "There is an odd number of Red Eyed Monks" 4. "There is an even number of Red Eyed Monks" 5. "There is more than one Red Eyed Monk"

WHERE'S THE FATHER?

The mother is 21 years older than the child. In 6 years from now, the mother will be 5 times as old as the child. Question: Where's the father?

file:///C|/Documents%20and%20Settings/Administrator/...ocuments/My%20Notepad/Jayesh%20placment/medium.shtml (7 of 32)8/7/2008 3:24:44 PM

[ wu :: riddles(medium) ]

Compare the numbers 0.99999... (infinitely many 9s) and 1. Which of the following statements is true? Why? 0.99999 ... < 1 0.99999 ... = 1 0.99999 ... > 1

.999 ...

Forum thread: Click here. Check out Icarus's (our local mathematician) insightful exposition on this problem.

Imagine a disk spinning like a record player turn table. Half of the

COLORED DISK SPIN SENSORS

disk is black and the other is white. Assume you have an unlimited number of color sensors. How many sensors would you have to place around the disk to determine the direction the disk is spinning? Where would they be placed?

a man has a gold chain with 7 links. he needs the service of a laborer for 7 days at a fee of one gold link per day. however, each day of work needs to be paid for separately. in other words, the worker must be paid each day after working and if the laborer is

GOLD CHAIN

ever overpaid he will quit with the extra money. also he will never allow himself to be owed a link. what is the fewest number of cuts to the chain to facilitate this arrangement and how does that guarantee payment?

DUMB CS JOKE M

Why do computer programmers always get Christmas and Halloween mixed up?

Using only a 5-gallon bucket and a 3-gallon bucket, put exactly

WATER BUCKETS

four gallons of water in the 5-gallon bucket. (Assume you have an infinite supply of water. No measurement markings on the buckets.)

file:///C|/Documents%20and%20Settings/Administrator/...ocuments/My%20Notepad/Jayesh%20placment/medium.shtml (8 of 32)8/7/2008 3:24:44 PM

[ wu :: riddles(medium) ]

This is the logic game of Mastermind. If you haven't played it before, here's how it works. There is a board that is sectioned off into many rows, each row having four slots in which pegs can be inserted. There are six different colors of pegs: green, red, yellow, brown, dark-blue, light-blue. There are two players, A and B. First, A makes up some arrangement of four pegs along a row, the colors and ordering of which are his or her choice. Then B spends the rest of the game trying to guess what A's arrangement is. For every guess that B makes, A will respond by putting some black and/or white pegs right next to A's guess; the black and white pegs are interpreted as follows: Black keypeg = one of B's pegs is the correct color and in the correct position
q

White keypeg = one of the B's pegs is the correct color but in the wrong position

MASTERMIND I

So if B manages to guess all four colors and positions correctly, A will respond with four black keypegs, and the game is over. The goal is to determine A's secret arrangement in the minimum number of guesses. Below, we see a completed game of Mastermind. Apparently the player was able to determine A's arrangement by using only four guesses. What's is A's arrangement?

TWO-CHILD FAMILY I

In a two-child family, one child is a boy. What is the probability that the other child is a girl?

file:///C|/Documents%20and%20Settings/Administrator/...ocuments/My%20Notepad/Jayesh%20placment/medium.shtml (9 of 32)8/7/2008 3:24:44 PM

[ wu :: riddles(medium) ]

TWO-CHILD FAMILY II

In a two-child family, the older child is a boy. What is the probability that the other child is a girl?

"Proof" that 2 = 1: a=b a2 = ab a2 - b2 = ab-b2 (a-b)(a+b) = b(a-b)

2=1

a+b = b b+b = b 2b = b 2=1

Does this argument make sense?

Four people, A, B, C, and D, are on one side of a bridge, and they all want to cross the bridge. However, it's late at night, so you can't cross without a flashlight. They only have one flashlight. Also, the bridge is only strong enough to support the weight of two people at once. The four people all walk at different speeds: A takes 1

BRIDGE CROSSING

minute to cross the bridge, B takes 2 minutes, C takes 5 minutes, and D takes 10 minutes. When two people cross together, sharing the flashlight, they walk at the slower person's rate. How quickly can the four cross the bridge? Note: Supposedly a classic Microsoft question.

file:///C|/Documents%20and%20Settings/Administrator/...ocuments/My%20Notepad/Jayesh%20placment/medium.shtml (10 of 32)8/7/2008 3:24:44 PM

[ wu :: riddles(medium) ]

If you drew a dot on the edge of a wheel and traced the path of the dot as the wheel rolled one complete revolution along a line, then the path formed would be called a cycloid (shown in red below), combining both forward and circular motion. What is the length of the path formed by one complete revolution? Assume the wheel has a radius of 1.

CYCLOID M

Ten people land on a deserted island. There they find lots of coconuts and a monkey. During their first day they gather coconuts and put them all in a community pile. After working all day they decide to sleep and divide them into ten equal piles the next morning. That night one castaway wakes up hungry and decides to take his share early. After dividing up the coconuts he finds he is one coconut short of ten equal piles. He also notices the monkey holding one more coconut. So he tries to take the monkey's coconut to have a total evenly divisible by 10. However when he tries to take it the monkey conks him on the head with it and kills him. Later another castaway wakes up hungry and decides to take his share early. On the way to the coconuts he finds the body of the first castaway, which pleases him because he will now be entitled to 1/9 of the total pile. After dividing them up into nine piles he is again one coconut short and tries to take the monkey's coconut. Again, the monkey conks the man on the head and kills him. One by one each of the remaining castaways goes through the same process, until the 10th person to wake up gets the entire pile for himself. What is the smallest number of possible coconuts in the pile, not counting the monkeys?

COCONUT MONKEY

file:///C|/Documents%20and%20Settings/Administrator/...ocuments/My%20Notepad/Jayesh%20placment/medium.shtml (11 of 32)8/7/2008 3:24:44 PM

[ wu :: riddles(medium) ]

You and your partner in crime are both arrested and questioned separately. You are offered a chance to confess, in which you agree to testify against you partner, in exchange for all charges being dropped against you, unless he testifies against you also. Your lawyer, whom you trust, says that the evidence against both of you, if neither confesses, is scant and you could expect to take a plea and each serve 3 years. If one implicates the other, the other can expect to serve 20 years. If both implicate each other you could each expect to serve 10 years. You assume the probability of your partner confessing is p. Your highest priority is to keep yourself out of the pokey, and your secondary motive is to keep you partner out. Specifically you are indifferent to you serving x years and your partner serving 2x years. At what value of p are you indifferent to confessing and not confessing?

PRISONER DILEMMA REDUX

Find the side length of the internal square and the radii of the internal circles, in terms of a.

file:///C|/Documents%20and%20Settings/Administrator/...ocuments/My%20Notepad/Jayesh%20placment/medium.shtml (12 of 32)8/7/2008 3:24:44 PM

[ wu :: riddles(medium) ]

JAPANESE TEMPLE GEOMETRY I

Note: This geometry problem comes from the tradition of sangaku. During the period between 1639 and 1854, many Japanese samurai apparently adopted geometry as a serious mental discipline! They would solve many difficult geometric problems, and carefully inscribe the solutions into beautiful wooden tablets, often with color. Then they would hang their solutions under roofs for public viewing, either to show respect for the elegance of math, or perhaps just to show off their intelligence and challenge other practitioners of sangaku -- a Japanese word that literally means mathematical tablet. Sangaku problems are usually just Euclidean geometry, but others seem almost impossible to do without cheating and using higher level math (e.g. calculus, affine transformations). Also notable is a strong emphasis on ellipses and circles, an emphasis not found in Western studies of geometry. Note 2: For a hardcopy of the above image only, click here and print. Hardcopies are useful for drawing lines and labeling vertices and what not.

file:///C|/Documents%20and%20Settings/Administrator/...ocuments/My%20Notepad/Jayesh%20placment/medium.shtml (13 of 32)8/7/2008 3:24:44 PM

[ wu :: riddles(medium) ]

Problem source: Fukagawa, H. and D. Pedoe. Japanese Temple Geometry problems. Charles Babbage Research Foundation: Winnipeg, 1989.

Find the radii of the internal circles, in terms of a.

JAPANESE TEMPLE GEOMETRY II

Note: For a hardcopy of the above image only, click here and print. Hardcopies are useful for drawing lines and labeling vertices and what not. Note 2: Thanks a lot to Jasvir Nagra for sending me such a clear solution, even with diagrams drawn with Postscript! Problem source: Fukagawa, H. and D. Pedoe. Japanese Temple Geometry problems. Charles Babbage Research Foundation: Winnipeg, 1989.
file:///C|/Documents%20and%20Settings/Administrator/...ocuments/My%20Notepad/Jayesh%20placment/medium.shtml (14 of 32)8/7/2008 3:24:44 PM

[ wu :: riddles(medium) ]

Find the radii of the internal circles, in terms of a.

JAPANESE TEMPLE GEOMETRY III

Note: For a hardcopy of the above image only, click here and print. Hardcopies are useful for drawing lines and labeling vertices and what not. Problem source: Fukagawa, H. and D. Pedoe. Japanese Temple Geometry problems. Charles Babbage Rs secretly loaded one of the dice, so that it will always come up 5. This increases his chances of winning to 2/3. Having learned of his evil deed, you're going to secretly load his other die so as to minimize his chance of winning. With what probability should you load each of the six faces? And how does that change his probability of winning?

file:///C|/Documents%20and%20Settings/Administrator/...ocuments/My%20Notepad/Jayesh%20placment/medium.shtml (15 of 32)8/7/2008 3:24:44 PM

[ wu :: riddles(medium) ]

Note: Writing credits to Matt Lahut.

There are three one-dimensional tracks, of length 12, 7, and 5 spaces respectively. You start with pennies in the first space of each track; your opponent starts with pennies in the last space of each track. On your turn, you may move any one of your pennies any number of spaces in either direction along a track (as a chess rook), however you are not permitted to bypass the other player's penny or occupy its space. If a player has no legal move, he loses.

PENNY TRACK GAME

What should your first move be?

Note: Hardcore puzzlers will probably solve this immediately, but newer people might take quite some time. For the newer people, the solution to this is a useful trick to have in your random-puzzlesolving arsenal. As a sub-problem, you may want to try solving it without the 5-length track.

You have an m x n grid with some real numbers in each cell. You can multiply any row by -1, or any column by -1. Show that by

POSITIVE MATRIX SUMS

making multiplications of this kind, the sum of the numbers in every row, and the sum of the numbers in every column, can all be made non-negative simultaneously.

file:///C|/Documents%20and%20Settings/Administrator/...ocuments/My%20Notepad/Jayesh%20placment/medium.shtml (16 of 32)8/7/2008 3:24:44 PM

[ wu :: riddles(medium) ]

Shown to the below-left is a checkerboard mutilated by the removal of two squares from each of two opposite corners. Shown to the below-right are two T-shaped tiles, each of which can cover four squares of the checkerboard exactly.

TETRAMINOES ON A CHECKERBOARD

If tiles of both kinds are abundant, and if tiles may be rotated, can the mutilated checkerboard be covered exactly with nonoverlapping tiles that match the colors of covered squares? Why?

In an presidential election between Willywutang and Billybutane, the winning candidate Willywutang has received n+k votes, whereas Billybutane has received n votes. (n and k are positive integers.) If ballots are counted in a random order, what is the

ALWAYS AHEAD

probability that Willywutang's accumulating count will always lead his opponent's, and why? Note: Cute eh? Also a practical calculation ... well, maybe.

file:///C|/Documents%20and%20Settings/Administrator/...ocuments/My%20Notepad/Jayesh%20placment/medium.shtml (17 of 32)8/7/2008 3:24:44 PM

[ wu :: riddles(medium) ]

A deck of 26 red and 26 black cards is shuffled into random order and placed face down. Then the cards are turned up one by one and observed by a guesser. He gets one guess: At a moment of his choice he may assert that the next card will turn up red. After this card is turned up the game ends and he wins if his assertion was correct, loses otherwise. And if he doesn't guess at all by the time all cards have been dealt, he loses by default. What guessing policy chosen in advance maximizes his chance of winning?

CALLING A CARD'S COLOR

Consider computing the product of two complex numbers (a + bi) and (c + di). By foiling the polynomials as we learned in grade school, we get

a c

+ bi + di

---------adi - bd ca + cbi ----------------

COMPLEX MULTIPLICATION

(ca - bd) + (ad + cb)i

Note that this standard method uses 4 multiplications and 2 additions to compute the product. (The plus sign in between (ca bd) and (ad + cb)i does not count as an addition. Think of a complex number as simply a 2-tuple.) It is actually possible to compute this complex product using only 3 multiplications and 3 additions. From a logic design perspective, this is preferable since multiplications are more expensive to implement than additions. Can you figure out how to do this?

file:///C|/Documents%20and%20Settings/Administrator/...ocuments/My%20Notepad/Jayesh%20placment/medium.shtml (18 of 32)8/7/2008 3:24:44 PM

[ wu :: riddles(medium) ]

A little while ago, Willy Wutang was thinking about the Sprinkler and Pump puzzle. Willy, having a creative mind (or at least a welldeveloped sense of mischief) had a flash of inspiration, and came up with a very novel sprinkler design. However, before he can patent his new sprinkler, Willy must figure out what it does--his lawyer says it's a necessary part of the patent process.

CRAZY SPRINKLER
1. First of all, does the sprinkler turn? If so, in which direction does it turn? 2. Second, what do the jets of water do? In the picture above, the sprinkler is shown at the instant just before the jets of water collide. However, in the picture the sprinkler is being held still, so that even if it wants to turn, it cannot. 3. Third, does the behaviour of the jets of water depend on whether or not the sprinkler is allowed to rotate? Does it maybe depend on how fast the sprinkler rotates?

Note: Writing credits to James Fingas!

file:///C|/Documents%20and%20Settings/Administrator/...ocuments/My%20Notepad/Jayesh%20placment/medium.shtml (19 of 32)8/7/2008 3:24:44 PM

[ wu :: riddles(medium) ]

What row of numbers comes next? 1 11 21 1211

CONWAY SEQUENCE

111221 312211 13112221

Note: Mathematician John Conway spent considerable time studying this sequence.

Willy Wutang is writing to his overseas sweetheart. He writes a steamy love letter, seals the envelope, and then inscribes the following cryptic address on the front: Wood, S April

CRYPTIC ADDRESS

England

Strangely enough, the letter reaches its intended recipient. What is Willy's sweetheart's address?

Note: Writing credits to James Fingas :)

file:///C|/Documents%20and%20Settings/Administrator/...ocuments/My%20Notepad/Jayesh%20placment/medium.shtml (20 of 32)8/7/2008 3:24:44 PM

[ wu :: riddles(medium) ]

Willy Wutang is making a robot, and he has bought himself a set of 24 finger gauges. For those of you who have never seen a set of these, each gauge is a strip of metal about 1/2" wide and about one foot long. Each is a different thickness (0.001" up to 0.024"). Each has a hole in one end, and they are fastened together with a nut and bolt, in order of thickness. Willy's calculations indicate that, with the flame thrower attachment extended, he must adjust the spark plug to be 0.086" from the jellied gasoline nozzle. He pulls out his trusty finger gauge set, and realizes they only go up to 0.024". How can he accurately set the spark plug distance? If you've figured out how Willy measured 0.086", what is the smallest measurement (in thousandths of an inch) that Willy can't make using his set of feeler gauges? What is the smallest measurement (in thousandths of an inch) that Willy can't approximate to +/-0.001"?

FINGER GAUGES

Note: Writing credits to the prolific James Fingas.

An acrobat thief enters an ancient temple, and finds the following scenario: 1. The roof of the temple is 100 meters high. 2. In the roof there are two holes, separated by 1 meter. 3. Through each hole passes a single gold rope, each going all the way to the floor. 4. There is nothing else in the room. The thief would like to cut and steal as much of the ropes as he can. However, he knows that if he falls from height that is greater than 10 meters, he will die. The only thing in his possession is a knife. How much length of rope can the acrobat thief get? And how?

ACROBAT THIEF AND GOLD ROPES

file:///C|/Documents%20and%20Settings/Administrator/...ocuments/My%20Notepad/Jayesh%20placment/medium.shtml (21 of 32)8/7/2008 3:24:44 PM

[ wu :: riddles(medium) ]

SUM OF RECIPROCALS

Show that the sum of the reciprocals of the first n positive numbers is not an integer for n > 1.

Willywutang is holding a birthday party. Either p or q people are going to show up, where p and q are relatively prime numbers. Willy wants to cut the cake beforehand, so his guests will not waste time waiting for slices to be carved out. What is the smallest number of slices Willy must cut out such that every guest can be given the same amount of cake, regardless of whether p or q people show up? The slices do not have to be equally sized, and each guest could receive more than one slice. Also, Willy is not allowed to eat the whole cake and give nothing to all the guests. Also, the entire cake must be distributed amongst the guests -there can be no leftovers.

CAKE DIVISION

Note 1: Two numbers are relatively prime if they share no common factors. So for example, the numbers 4 and 9 are relatively prime, but the numbers 10 and 15 are not because they share the number 5 as a common factor. Note 2: (5/27/2003 2:17PM) Edited to specify that the entire cake must be given away.

1/27/2003 8:30PM Somewhere in Northern Eurasia, a group of 20 lemmings is planning a special group suicide this year. Each of the lemmings will be placed in a random position along a thin, 100 meter long plank of wood which is floating in the sea. Each lemming is equally likely to be facing either end of the plank. At time t=0, all the lemmings walk forward at a slow speed of 1 meter per minute. If a lemming bumps into another lemming, the two both reverse directions. If a lemming falls off the plank, he drowns. What is the longest time that must elapse till all the lemmings have drowned?

LEMMING DROWNINGS

file:///C|/Documents%20and%20Settings/Administrator/...ocuments/My%20Notepad/Jayesh%20placment/medium.shtml (22 of 32)8/7/2008 3:24:44 PM

[ wu :: riddles(medium) ]

Author: William Wu Note 1: If you make a certain observation, the calculations required become very trivial! Note 2: Yes, I know Lemmings don't really commit group suicides. Problem and storyline was inspired by the Lemmings video game.

1/27/2003 8:30PM Imagine a cube where each edge is a 1 ohm resistor. Find the

RESISTOR CUBE

resistance between opposite corners of the cube. There are many ways to solve this problem, but some ways are more clever than others.

1/27/2003 8:30PM Two astronauts are standing on a spinning space station shaped like a disk. They are the same radial distance away from the disk's center, and standing opposite to each other across from the center (e.g., if you draw a line connecting the two astronauts, the line crosses the disk's center.) One astronaut wants to toss a wrench to the other. Among the infinitude of trajectories which will accomplish this goal, characterize one of the trajectories without writing a single equation.

ASTROWRENCH P

Note: Be wary grasshopper. There are several common wrong answers to this problem!

file:///C|/Documents%20and%20Settings/Administrator/...ocuments/My%20Notepad/Jayesh%20placment/medium.shtml (23 of 32)8/7/2008 3:24:44 PM

[ wu :: riddles(medium) ]

2/2/2003 3:31AM What are the next few terms in the sequence? /-\ /-\//-\\ /-\-/-\/-\ /-\---

SLASH AND DASH SEQUENCE

///-\\\ //-\\/-\-/-\ Hint 1: Maybe it'll go easier if you replace - with 0, and the / and \ with ( and ). Or not. Hint 2: These are just the numbers 1 through 10, expressed differently. And despite the presence of zero, you can't express zero this way. Hint 3: All your base are belong to prime numbers.

2/2/2003 3:31AM Two thieves conspire to steal a valuable necklace made of diamonds and rubies (evenly spaced, but not necessarily alternating or symmetric). After they take it home, they decide that the only way to divide the booty fairly is to physically cut the necklace in half. Prove that, if there is an even number of diamonds and an even number of rubies, it's possible to cut the necklace into two pieces, each of which contains half the diamonds and half the rubies.

JEWEL THIEVES

file:///C|/Documents%20and%20Settings/Administrator/...ocuments/My%20Notepad/Jayesh%20placment/medium.shtml (24 of 32)8/7/2008 3:24:44 PM

[ wu :: riddles(medium) ]

3/9/2003 4:11AM In a cliche effort to illustrate the importance of teamwork-oriented problem solving, the Boss has chained Dilbert to Carol The Secretary via wire wrapped around their wrists, as shown in the following snapshot:

WIRECUFFS

The goal is for Dilbert and Carol to unlink themselves from each other; considering what a horrible woman Carol is, Dilbert wouldn't have it any other way. The wire is unbreakable, and as much as Dilbert would like to saw off Carol's limbs, that's against company policy. How can Dilbert and Carol get away from each other?

4/7/2003 12:47AM While recovering from wounds in the American Civil War, a Captain Fox threw needles at a surface ruled with parallel lines, and was thus able to experimentally infer the value of pi. True story! To find out exactly how pi plays into this scenario, solve the following problem: A surface is ruled with parallel lines. The lines are at distance D apart from each other. Suppose we throw a needle of length L on the surface at random. What is the probability that the needle will intersect one of the lines?

BUFFON'S NEEDLE M

Note 1: A famous problem posed and solved in 1777 by French naturalist Buffon. It has long since fascinated scientists, and marks the origin of geometrical probability -- the analysis of geometrical configurations of randomly placed objects. Note 2: These 19th century experiments began development of the

file:///C|/Documents%20and%20Settings/Administrator/...ocuments/My%20Notepad/Jayesh%20placment/medium.shtml (25 of 32)8/7/2008 3:24:44 PM

[ wu :: riddles(medium) ]

Monte Carlo method, which uses repeated simulation to approximate true statistics. Note 3: This isn't much of a riddle ... more like just an interesting mathematical exercise. The most challenging part is setting up the problem.

4/7/2003 12:47AM Samwise Gamgee has a square plot of land, each side being 1 unit. One day, Sam finds out that the dark Lord Sauron has a telephone line that he uses to speak with a traitor amongst the hobbits. Gandalf informs him that the telephone line runs in a straight line parallel to the ground and passes beneath the square plot of land, but he does not know its location. Sam decides to dig up around the perimeter of his land to discover the telephone line, but Gandalf says it is not necessary to dig around the entire length of 4 units. Sam brightens up, and says "I know what you mean. I can just dig 3 sides and still discover it. For even if the phone line runs along the fourth side, I will still detect it at the end points ! " Gandalf shakes his head. "No, Sam. You are on the right track, but you can do better than that." What solution does Gandalf have in mind for the optimum length of the "digging curve" ?

SAMWISE AND GANDALF

file:///C|/Documents%20and%20Settings/Administrator/...ocuments/My%20Notepad/Jayesh%20placment/medium.shtml (26 of 32)8/7/2008 3:24:44 PM

[ wu :: riddles(medium) ]

4/7/2003 12:47AM

RETROGRADE CHESS I C

What was the last move made?

Note: From Karl Fabel's 1955 book Rund um das Schachbrett.

5/11/2003 10:44PM

Consider an N x N grid. Denote one corner as point A, and the

PATH COLLISION

opposite corner as point B. George is walking from A to B, and Lennie is walking from B to A. All paths are equally likely, as long as they follow the grid and never move away from the destination. (Hence George's path can never move down or left, and Lennie's path can never move up or right.)

file:///C|/Documents%20and%20Settings/Administrator/...ocuments/My%20Notepad/Jayesh%20placment/medium.shtml (27 of 32)8/7/2008 3:24:44 PM

[ wu :: riddles(medium) ]

What is the probability that George and Lennie collide? If George runs and thus moves three times faster than Lennie, what is the probability of collision?

5/11/2003 10:44PM Assume the Earth is a perfect sphere of radius r and suppose a

ROPE AROUND THE EARTH

rope of zero elasticity is tied tightly around it. One metre is now added to the rope's length. If the rope is now pulled at one point as high as possible above the Earth's surface, what height will be reached?

5/11/2003 10:44PM Two urns contain the same total numbers of balls, some blacks and some whites in each. From each urn are drawn n balls with replacement, where n >= 3. Find the number of drawings and the composition of the two urns so that the probability that all white

MOLINA'S URNS

balls are drawn from the first urn is equal to the probability that the drawing from the second is either all whites or all blacks.

Author: E.C. Molina.

5/27/2003 2:24PM Consider a gambling machine A. When you put in $X and pull the handle, it will spit out (equally likely) either $0.7*X, $0.8*X, $0.9*X, $1.1*X, $1.2*X, or $1.5*X. Now consider the following two ways of playing this machine:

TWO GAMBLING MACHINES

1. Put in $1, pull the handle, and keep whatever you get. Repeat. 2. Initially, put in $1. Pull the handle, then put in whatever you

file:///C|/Documents%20and%20Settings/Administrator/...ocuments/My%20Notepad/Jayesh%20placment/medium.shtml (28 of 32)8/7/2008 3:24:44 PM

[ wu :: riddles(medium) ]

get. Repeat. Can you win money with this machine? Which is the better way to play? How can this be?

5/11/2003 10:44PM

HEXAGON IN CIRCLE

A hexagon with sides of length 2, 7, 2, 11, 7, 11 is inscribed in a circle. Find the radius of the circle.

Contributor: Mark Newheiser

5:15 PM 8/3/2004

You have 1000 pirates, who are all extremely greedy, heartless, and perfectly rational. They're also aware that all the other pirates share these characteristics. They're all ranked by the order in which they joined the group, from pirate one down to a thousand. They've stumbled across a huge horde of treasure, and they have to decide how to split it up. Every day they will vote to either kill the lowest ranking pirate, or split the treasure up among the surviving pirates. If 50% or more of them vote to split it, the treasure gets split. Otherwise, they kill the lowest ranking pirate and repeat the process until half or more of the pirates decide to split the treasure. The question, of course, is at what point will the treasure be split, and what will the precise vote be? After that, consider solving the problem when a two-thirds or three-fourths majority is required. Try to generalize the result.

ELEGANTLY GREEDY PIRATES

file:///C|/Documents%20and%20Settings/Administrator/...ocuments/My%20Notepad/Jayesh%20placment/medium.shtml (29 of 32)8/7/2008 3:24:44 PM

[ wu :: riddles(medium) ]

Contributor: William Wu

9:00 PM 8/19/2004

Starting with 0 dollars, you repeatedly flip a fair coin, earning $1 each time heads appears, and losing $1 each time tails appears. When your net cash reaches either A or -B, you stop gambling. 1. As a function of A and B, compute the expected number of flips until the game stops. 2. Now consider the unstopped version of this process, in which

FAIR COIN GAMBLING

you gamble indefinitely regardless of your current profit or debt. Prove that the expected time till your net cash is +$1 is infinite. Likewise, the expected time till your net cash is -$1 is also infinite. And yet one of these two events must occur upon the first coin flip!

Forum threads: More Heads Than Tails and Lazy hunter catch the rabbit.

Contributor: William Wu

9:00 PM 8/19/2004

Imagine a upright ladder with n rungs. At each time step, a frog jumps to one of the n rungs uniformly at random. (The frog could jump in place.) Assume time starts counting from 1, and the initial rung position is also chosen uniformly at random.

JUMPING FROG ON LADDER

As the number of rungs tends to infinity, what is the expected time till the frog first jumps downwards? Prove it.

Author: William Wu

file:///C|/Documents%20and%20Settings/Administrator/...ocuments/My%20Notepad/Jayesh%20placment/medium.shtml (30 of 32)8/7/2008 3:24:44 PM

[ wu :: riddles(medium) ]

Contributor: Eigenray

9:00 PM 8/19/2004

Let Q denote the set of rationals. How many colors are required to color every point of Q^k in such a way that no two points a unit

COLORING THE RATIONAL PLANE

distance apart are the same color? 1. Easy: k = 1 2. Medium: k=2,3 3. Hard: k>3

Contributor: Joc Koelman

9:00 PM 8/19/2004

RANDOM POINTS ON SPHERE

Two points on the surface of a sphere are drawn uniformly at random. What is the maximum likelihood estimate of the distance between these two points? The answer may be shocking at first. After getting the answer, try to explain it intuitively.

Contributor: Aryabhatta

9:00 PM 8/19/2004

You are given n>=2 points in the 2-D plane. For each pair of points (P,Q) from the n points, mark the midpoint of PQ red. Show that there are at least 2n-3 distinct red points. For each n, show an

NUMBER OF MIDPOINTS

arrangement of n points for which there are exactly 2n-3 distinct red points.

Source: Iranian Mathematical Olympiad

file:///C|/Documents%20and%20Settings/Administrator/...ocuments/My%20Notepad/Jayesh%20placment/medium.shtml (31 of 32)8/7/2008 3:24:44 PM

[ wu :: riddles(hard) ]

RECENT ADDITIONS
Check out latest puzzles by perusing the forum and the 10 most recent posts. Latest additions to cover site can be seen by clicking here.

relatively hard
An evil king has 1000 bottles of wine. A neighboring queen plots to kill the bad king, and sends a servant to poison the wine. The king's guards catch the servant after he has only poisoned one bottle. The guards don't know which bottle was poisoned, but they do know that the poison is so potent that even if it was diluted 1,000,000 times, it would still be fatal. Furthermore, the effects of the poison take one month to surface. The king decides he will get some of his prisoners in his vast dungeons to drink the wine. Rather than using 1000 prisoners each assigned to a particular bottle, this king knows that he needs to murder no more than 10 prisoners to figure out what bottle is poisoned, and will still be able to drink the rest of the wine in 5 weeks time. How does he pull this off?

CRIMINAL CUPBEARERS

Sheila and He-Man are twins; Sheila is the OLDER twin. Assume they were born immediately after each other, an infinitesimally small - but nonzero - amount of time apart. During one year in the course of their lives, Sheila celebrates her birthday two days AFTER He-Man does. How is this possible? 10/28/2002 3:58AM Bonus: What is the maximum amount of time by which Sheila and He-Man can be apart in their birthday celebrations during the same year? I think it's more than two days.

BIRTHDAY TWINS
Note: For both Sheila and He-Man, these birthday celebrations happen on the actual birthday date -- it cannot be a celebration that occurs at a date earlier or later than the actual birthday date for whatever reasons of convenience. Also, the solution has nothing to do with the theory of relativity or any other over complicated nonsense like that.

file:///C|/Documents%20and%20Settings/Administrator/My%20Documents/My%20Notepad/Jayesh%20placment/hard.shtml (2 of 69)8/7/2008 3:24:49 PM

[ wu :: riddles(hard) ]

There is a circular jail with 100 cells numbered 1-100. Each cell has an inmate and the door is locked. One night the jailor gets drunk and starts running around the jail in circles. In his first round he opens each

CIRCULAR JAIL CELL

door. In his second round he visits every 2nd door (2,4,6---) and shuts the door. In the 3rd round he visits every 3rd door (3,6,9---) and if the door is shut he opens it, if it is open he shuts it. This continues for 100 rounds (i.e. 4,8,12 ---; 5,10,15 ---; ---; 49,98 etc.) and exhausted the jailor falls down. How many prisoners found their doors open after 100 rounds?

100 prisoners are imprisoned in solitary cells. Each cell is windowless and soundproof. There's a central living room with one light bulb; the bulb is initially off. No prisoner can see the light bulb from his or her own cell. Each day, the warden picks a prisoner equally at random, and that prisoner visits the central living room; at the end of the day the prisoner is returned to his cell. While in the living room, the prisoner can toggle the bulb if he or she wishes. Also, the prisoner has the option of asserting the claim that all 100 prisoners have been to the living room. If this assertion is false (that is, some prisoners still haven't been to the living room), all 100 prisoners will be shot for their stupidity. However, if it is indeed true, all prisoners are set free and inducted into MENSA, since the world can always use more smart people. Thus, the assertion should only be made if the prisoner is 100% certain of its validity. Before this whole procedure begins, the prisoners are allowed to get together in the courtyard to discuss a plan. What is the optimal plan they can agree on, so that eventually, someone will make a correct assertion?

Note 1: (1/8/2003 3:51PM Update) What is meant by optimal? If your solution is optimal, it means you can prove that no other algorithm can produce a lower average running time. This is usually very hard to do though, and I would be surprised if anyone ever sends me such a proof. So the best we can do in the meantime is try to beat the best average running time we know of. The number to beat so far is around 3500 days. So BEFORE YOU E-MAIL ME YOUR SOLUTION, check its average time to see if beats the 4000 day ballpark. If you get a number around 27-28 years, then you've found the solution most people who solve the puzzle come up with. However, it's not optimal. Note 2: (1/8/2003 3:49PM Update) How to compute average running time? The preferred method is to do a probabilistic analysis using pencil and paper. But if you haven't learned about stuff like that, a much simpler way is to just program your solution and run it maybe 100 times, recording how many days elapsed in each invocation. Afterwards you should have an array of 100 numbers. Now take the average
file:///C|/Documents%20and%20Settings/Administrator/My%20Documents/My%20Notepad/Jayesh%20placment/hard.shtml (3 of 69)8/7/2008 3:24:49 PM

100 PRISONERS AND A LIGHT BULB >=P

[ wu :: riddles(hard) ]

of all them, and you'll have an empirical average which is close to the theoretical one. Note 3: (11/7/2002 7:35AM Update) The problem statement used to say "The prisoners are allowed to get together one night to discuss a plan." In the forum, quite a few people mentioned the clever solution of simply having the planning meeting in the central living room, and then asserting that everyone has been there on the first day of the random selection process. To assure that this problem is not so easily defeated, I have stipulated that the meeting happen in the courtyard. Note 4: Does anyone know where this riddle originated? I got it from a friend who got it from another friend who doesn't know where it comes from. E-mail me at wwu at ocf.berkeley.edu. Note 5: (1/1/2003 1:25AM Update) Edited to add that the cells are soundproof and windowless. Note 6: (1/21/2003 1:03AM Update) Edited to clarify that chosen prisoners are turned to their cells at the end of the day. Note 7: (2/6/2003 11:27PM Update) Edited to clarify that the meeting must happen before the whole ordeal begins.

Using all five of the pieces shown below, make a new square.

SQUARE FORMATION

file:///C|/Documents%20and%20Settings/Administrator/My%20Documents/My%20Notepad/Jayesh%20placment/hard.shtml (4 of 69)8/7/2008 3:24:49 PM

[ wu :: riddles(hard) ]

Note: Click here and print out the image. Then cut out the pieces and play with them on your desk.

this is a magic trick performed by two magicians, A and B, with one regular, shuffled deck of 52 cards. A asks a member of the audience to randomly select 5 cards out of a deck. the audience member -- who we will refer to as C from here on -- then hands the 5 cards back to magician A. after looking at the 5 cards, A picks one of the 5 cards and gives it back to C. A then arranges the other four cards in some way, and gives those 4 cards face down, in a neat pile, to B. B looks at these 4 cards and then determines what card is in C's hand (the missing 5th card). how is this trick done?

5 CARD MAGIC TRICK M


Note 1: There's no secretive message communication in the solution, like encoded speech or ninja hand signals or ESP or whatever ... the only communication between the two magicians is in the logic of the 4 cards transferred from A to B. Think of these magicians as mathematicians.

file:///C|/Documents%20and%20Settings/Administrator/My%20Documents/My%20Notepad/Jayesh%20placment/hard.shtml (5 of 69)8/7/2008 3:24:49 PM

[ wu :: riddles(hard) ]

Note 2: After you've figured this one out, try 5 CARD MAGIC TRICK REDUX for the next level of difficulty. Note 3: This magic trick is originally credited to magician and mathematician Fitch Cheney.

you're a cyborg in a pistol duel with two other cyborgs. you have been programmed to fire pistols with an accuracy of 33%. the other two cyborgs shoot with accuracies of 100% and 50%, respectively. the rules of the duel are one shot per-cyborg per-round. the shooting order is from worst shooter to best shooter. thus, you go first, the 50% guy goes second, and the 100% guy goes third; repeat. if a cyborg dies, we just skip his or her turn, obviously. what should you shoot at in round 1 to maximize your chances of survival over time?

THREE-WAY PISTOL DUEL

a corporate business man has two cubes on his office desk. every day he arranges both cubes so that the front faces show the current day of the month. what numbers are on the faces of the cubes to allow this?

CALENDAR CUBES I

Note: You can't represent the day "7" with a single cube with a side that says 7 on it. You have to use both cubes all the time. So the 7th day would be "07". I also should note that this is a really sly problem. Almost unfair.

At a fork in the road between two cities, you see 2 people. One always tells the truth, and comes from the city of safety. The other person always lies and comes from the city of cannibals, where they will eat you. They both look exactly the same. You must choose one of the persons, and ask him one and only one

FORK IN THE ROAD I

question (no compound questions either, such as "is this shirt red and which way to safety?"). What question could you ask to find out which path leads to the city of safety? Big Hint: The answer is a very common question.

file:///C|/Documents%20and%20Settings/Administrator/My%20Documents/My%20Notepad/Jayesh%20placment/hard.shtml (6 of 69)8/7/2008 3:24:49 PM

[ wu :: riddles(hard) ]

A traveler, on his way to a certain village A, reaches a road junction, where he can turn left or right. He knows that only one of the two roads leads to village A, but unfortunately, he does not know which one. Fortunately, he sees two twin-brothers standing at the road junction, and he decides to ask them for directions. The traveler knows that one of the two brothers always tells the truth and the other one always lies. Unfortunately, he does not know which one always tells the truth and which one always lies. How can the traveler find out the way to village A by asking just one question to one of the two brothers?

FORK IN THE ROAD II

Note 1: Much harder than the more commonly known FORK IN THE ROAD I. Yes, they are different problems. It's worth noting that a solution to FORK IN THE ROAD II will work equally well on FORK IN THE ROAD I; however, not necessarily the converse. Note 2: Actually, there's yet another solution which solves both I and II equally well, and it is not that difficult to come up with.

you have two eggs. you need to figure out how high an egg can fall from a 100 story building before it breaks. the eggs might break from the first floor, or might even survive a drop from the 100th floor -- you have no a priori information. what is the largest number of egg drops you would ever have to do to find the right floor? (i.e. what's the most efficient way to drop the eggs and determine an answer?) you are allowed to break both eggs, as long as you identify the correct floor afterwards. after you've solved the above problem, generalize. define the "break floor" as the lowest floor in a

EGG DROPPING

building from which an egg would break if dropped. given an n story building and a supply of d eggs, find the strategy which minimizes (in the worst case) the number of experimental drops required to determine the break floor.

Note: Interestingly, the solution is similar to a commercial algorithm used for stress-testing the reliability of TCP/IP networks. Got this from Spring 2002 CS170, taught by Dr. Satish Rao.

file:///C|/Documents%20and%20Settings/Administrator/My%20Documents/My%20Notepad/Jayesh%20placment/hard.shtml (7 of 69)8/7/2008 3:24:49 PM

[ wu :: riddles(hard) ]

A pirate ship captures a treasure of 1000 golden coins. The treasure has to be split among the 5 pirates: 1, 2, 3, 4, and 5 in order of rank. The pirates have the following important characteristics: infinitely

GREEDY PIRATES

smart, bloodthirsty, greedy. Starting with pirate 5 they can make a proposal how to split up the treasure. This proposal can either be accepted or the pirate is thrown overboard. A proposal is accepted if and only if a majority of the pirates agrees on it. What proposal should pirate 5 make?

Local Berkeley professors Dr. Demmel and Dr. Shewchuk bump into each other on Telegraph Ave. They haven't seen each other since Vietnam.

Shewchuk Demmel Shewchuk

hey! how have you been? great! i got married and i have three daughters now really? how old are they? well, the product of their ages is 72, and the sum of their ages is the same as the number on that building over there.. right, ok ... oh wait ... hmm, i still don't know oh sorry, the oldest one just started to play the piano wonderful! my oldest is the same age!

DAUGHTERS' AGES

Demmel

Shewchuk Demmel Shewchuk

How old are the daughters?

file:///C|/Documents%20and%20Settings/Administrator/My%20Documents/My%20Notepad/Jayesh%20placment/hard.shtml (8 of 69)8/7/2008 3:24:49 PM

[ wu :: riddles(hard) ]

10 straight-jacketed prisoners are on death row. Tomorrow they will be arranged in single file, all facing one direction. The guy in the front of the line (he can't see anything in front of him) will be called the 1st guy, and the guy in the back of the line (he can see the heads of the other nine people) will be called the 10th guy. An executioner will then put a hat on everyone's head; the hat will either be black or white, totally random. Prisoners cannot see the color of their own hat. The executioner then goes to the 10th guy and asks him what color hat he is wearing; the prisoner can respond with either "black" or "white". If what he says matches the color of the hat he's wearing, he will live. Else, he dies. The executioner then proceeds to the 9th guy, and asks the same question, then asks the 8th guy ... this continues until all of

SINGLE-FILE HAT EXECUTION

the prisoners have been queried. This is the night before the execution. The prisoners are allowed to get together to discuss a plan for maximizing the number of lives saved tomorrow. What is the optimal plan? After you have solved the above problem, generalize. There are N prisoners and K different colors of hats. What's the optimal plan? Hint: if there are N prisoners, you can save N-1 lives, guaranteed!

Trianglia is a jacked-up island where no road has a dead end, and all the crossroads are "Y" shaped. The young prince of Trianglia mounts his horse, and is about to go on a quest to explore the land of Trianglia. He gets to the road by his palace, when the mother queen comes out and shouts: "But Charles, how will you find your way back?". "Don't worry Elizabeth", the prince replies, "I will turn right in every second crossroad to which I arrive, and left otherwise. Thus I shall surely return to the palace sooner or later." Is the prince right?

TRIANGLIA

file:///C|/Documents%20and%20Settings/Administrator/My%20Documents/My%20Notepad/Jayesh%20placment/hard.shtml (9 of 69)8/7/2008 3:24:49 PM

[ wu :: riddles(hard) ]

Four ghostly galleons call them E, F, G and H, sail on a ghostly sea so foggy that visibility is nearly zero. Each pursues its course steadily, changing neither its speed nor heading. G collides with H amidships; but since they are ghostly galleons they pass through each other with no damage nor change in course. As they part, Hs captain hears Gs say Damnation! Thats our third collision this night! A little while later, F runs into H amidships with the same effect (none) and Hs captain hears the same outburst from Fs. What can Hs captain do to avoid a third collision and yet reach his original destination, whatever it may be, and why will doing that succeed? Problem Source: Dr. William Kahan, Math H110 (honors linear algebra), UC Berkeley

FOUR GHOST SHIPS

ROTTEN APPLE M

An apple is in the shape of a ball of radius 31 mm. A worm gets into the apple and digs a tunnel of total length 61 mm, and then leaves the apple. (The tunnel need not be a straight line.) Prove that one can cut the apple with a straight slice through the center so that one of the two halves is not rotten.

You have 3,000 bananas and a camel which can carry at most 1,000 bananas at a time. The camel eats a banana before moving a unit. You want to transport the bananas 1,000 units. What is the maximum

CAMEL BANANA TRANSPORT

number of uneaten bananas that you can move 1,000 units? Problem source: 11th Grade Honors Precalculus, Dr. James J. Hogan High School

You are wearing a blindfold and thick gloves. An infinite number of quarters are laid out before you on a table of infinite area. Someone tells you that 20 of these quarters are tails and the rest are heads. He says that if you can split the quarters into 2 piles where the number of tails quarters is the same in both piles, then you win all of the quarters. You are allowed to move the quarters and to flip them over, but you can never tell what state a quarter is currently in (the blindfold prevents you from seeing, and the gloves prevent you from feeling which side is heads or tails). How do you partition the quarters so that you can win them all? Hint 1: Your algorithm should run in under 30 seconds. Hint 2: If an infinite number of quarters confuses you, try 100.
file:///C|/Documents%20and%20Settings/Administrator/My%20Documents/My%20Notepad/Jayesh%20placment/hard.shtml (10 of 69)8/7/2008 3:24:49 PM

INFINITE QUARTER SEQUENCE

[ wu :: riddles(hard) ]

Three men go to a cheap motel, and the desk clerk charges them a sum of $30.00 for the night. The three of them split the cost ten dollars each. Later the manager comes over and tells the desk clerk that he overcharged the men, since the actual cost should have been $25.00. The manager gives the bellboy $5.00 and tells him to give it to the men. The bellboy, however, decides to cheat the men and pockets $2.00, giving each of the men only one dollar. Now each man has paid $9.00 to stay for the night, and 3 x $9.00 = $27.00. The bellboy has pocketed $2.00. But $27.00 + $2.00 = $29.00. Where is the missing $1.00? WTF?

VANISHING DOLLAR

The Tortoise challenged the great warrior Achilles to a 100 meter foot race, claiming that he would win as long as Achilles granted him a little headstart. Achilles laughed, for he was a mighty warrior swift of foot, whereas the Tortoise was heavy and slow. "How long of a head start do you need?" asked Achilles, smiling. "Ten meters," said the Tortoise. Achilles laughs. "OK, you will most definitely lose, but we can race if you really want." "Actually, I will most definitely win, and I can prove it to you with a simple argument," said the Tortoise. "Go on then," Achilles replied, with less confidence than he felt before. He knew he was the superior athlete, but he also knew the Tortoise had the sharper wits, and he had lost many a bewildering argument with him before this. "Suppose," began the Tortoise, "that you give me a 10-meter head start. Would you say that you could cover that 10 meters between us very quickly?" "Very quickly," Achilles affirmed. "And in that time, how far should I have gone, do you think?" "Perhaps a meter - no more," said Achilles after a moment's thought. "Very well," replied the Tortoise, "so now there is a meter between us. And you would catch up that distance very quickly?" "Very quickly indeed!" "And yet, in that time I shall have gone a little way farther, so that now you must catch that distance up, yes?" "Ye-es," said Achilles slowly. "And while you are doing so, I shall have gone a little way farther, so that you must then catch up the
file:///C|/Documents%20and%20Settings/Administrator/My%20Documents/My%20Notepad/Jayesh%20placment/hard.shtml (11 of 69)8/7/2008 3:24:49 PM

ZENO'S PARADOX M

[ wu :: riddles(hard) ]

new distance," the Tortoise continued smoothly. Achilles said nothing. "And so you see, in each moment you must be catching up the distance between us, and yet I - at the same time - will be adding a new distance, however small, for you to catch up again." "Indeed, it must be so," said Achilles wearily. "And so you can never catch up," the Tortoise concluded sympathetically. "You are right, as always," said Achilles sadly - and conceded the race. Was it really impossible for Achilles to win the race? Explain. Note: Story adapted from Douglas Hofstaeder's awesome book, Godel, Escher, Bach.

We know that the derivative of x2 with respect to x is 2x. However, what if we rewrite x2 as the sum of x x's, and then take the derivative: d/dx[ x2 ] = d/dx[ x + x + x + ... (x times) ] = d/dx[x] + d/dx[x] + d/dx[x] ... (x times) = 1 + 1 + 1 + ... (x times)

DIFFERENTIATION DISASTER M

=x

This argument shows that the derivative of x2 with respect to x is actually x. So what's going on here? Note: Most people with some math experience can show that some part of the argument is erroneous. As in simply, something doesn't follow. However, a full solution will explain why this argument attacks something that lies at the very heart of calculus itself, and that is what really explains why it's erroneous. Forum thread: (spoilers) Click here

file:///C|/Documents%20and%20Settings/Administrator/My%20Documents/My%20Notepad/Jayesh%20placment/hard.shtml (12 of 69)8/7/2008 3:24:49 PM

[ wu :: riddles(hard) ]

PEOPLE COMBINATIONS ROOM

You have an empty room, and a group of people waiting outside the room. At each step, you may either get one person into the room, or get one out. Can you make subsequent steps, so that every possible combination of people is achieved exactly once?

100! contains how many trailing zeroes?

100 FACTORIAL

Note: For those not familiar with factorial, n! = n*(n-1)*(n-2) ... 2*1. So for example, 3! = 3*2*1 = 6.

"October 2, 2001" in MMDDYYYY format is a palindrome (a string that reads the same forwards as it does

PALINDROME DATES

backwards). Pretty cool, check it out: 10/02/2001 --> 10022001. When was the last date before October 2, 2001 that is also a palindrome?

You have an infinite cache of cigarettes. What is the maximum number of cigarettes you can place on a table so that every cigarette touches every other cigarette? (You can't bend the cigarettes.) Rigorously prove that your number is optimal.

Note 1: You can't just do a star configuration, like an asterisk. Remember that a cigarette has a thickness, and can't be treated as a line of infinitesimally small width. Thus, in an asterisk configuration of six cigarettes, there will be a hexagonal hole in the middle, and any cigarette would not be touching the cigarette opposite to it. Also, you can't just make an octagonal ring of cigarettes, because then each cigarette would only be touching the cigarettes directly to the right and left of it -- the question wants every cigarette to touch each other directly, not just be somehow indirectly connected to every other

CIGARETTES ON A TABLE >=P

cigarette. Note 2: Check out cigarettes.shtml for my analysis so far (spoiler warning: contains solution configurations for 6 and 7 cigarettes), and this bulletin board thread for what some others have thought about this problem. Note 3: Now that you have constructed an optimal configuration and proven it's optimality, generalize.

file:///C|/Documents%20and%20Settings/Administrator/My%20Documents/My%20Notepad/Jayesh%20placment/hard.shtml (13 of 69)8/7/2008 3:24:49 PM

[ wu :: riddles(hard) ]

Suppose you have an infinite cache of cylindrical widgets of height h and radius r. Derive a closed-form formula that calculates the number of widgets in the max clique configuration. And send the formula to me please =) I believe this can be done because this problem is referenced in this following Calvin University math assignment. Note 4: 3/9/2003 6:01AM A forum member informs that the great Martin Gardner solved the problem in one of his books; I have yet to check this out.

You have two 3-bit sensors, A and B, that measure the same thing, whatever it is -- temperature of the room, radioactivity levels, whatever. Both sensors are hooked up to the same CPU, which takes in the sensor readings. You know that the sensors are designed so that their readings can be off by at most one bit. We claim that if B knows that A has sent the CPU a 3-bit sequence, then B only needs to send 2 bits, and the CPU will be able to reconstruct B's 3-bit measurement, thereby conserving bandwidth. How is this so?

3-BIT SENSORS

Hint: See the maximum minimum Hamming distance riddle for some powerful ideas.

Note: This riddle is actually related to some signal processing research I'm doing at UC Berkeley. Credit to Dr. Kannan Ramchandran, my research professor.

alice sends different partial messages to a bunch of different receivers. by partial, we mean that one message by itself doesn't convey any meaningful information. let us denote the set of receivers as R. the messages are designed such that if any n receivers get together, they can somehow pool their partial messages together to get a meaningful message -- alice's intended message. however, if any n-1 or less

MESSAGE RECONSTRUCTION M

receivers get together, they can't reconstruct anything meaningful whatsoever. n < |R|. what kind of messages are being sent by alice, and what mathematical function do the receivers apply on their pooled partial data to determine the intended message? Note: If you solved this riddle about 30 years ago, you could've published a paper on it and earned a Ph. D. Unfortunately, someone's already done that. His full name is not very well known, but the initial of his last name is peppered across cyberspace.

file:///C|/Documents%20and%20Settings/Administrator/My%20Documents/My%20Notepad/Jayesh%20placment/hard.shtml (14 of 69)8/7/2008 3:24:49 PM

[ wu :: riddles(hard) ]

Consider a finite sequence of distinct integers. A subsequence is a sequence formed by deleting some items from the original sequence without disturbing their relative ordering. A subsequence is called monotone if it is either increasing (each term is larger than the one before it) or decreasing (each term is smaller than the one before it). For example, if the sequence is 4, 6, 3, 5, 7, 1, 2, 9, 8, 10, then 4, 6, 8, 10 is a monotone (increasing) subsequence of length 4 and 6, 5, 2 is a monotone (decreasing) subsequence of length 3. a) Find a sequence of 9 distinct integers that has no monotone subsequence of length 4. b) Show that every such sequence of length 10 has a monotone subsequence of length 4. c) Generalize. How long must the sequence be to guarantee a monotone subsequence of length n?

MONOTONIC SUBSEQUENCE M

I am thinking of an integer n with 0 <= n <= 15. To figure out what number I'm thinking of, you can ask

7 BOOLEAN QUESTIONS M

me 7 yes-or-no questions -- questions that can only be answered with either "yes" or "no". The questions must be independent of each other, their answers, and the order in which they are answered. (So you can't ask a question like, "if the answer to the previous question was "yes", then is n larger than 10, otherwise is n even?") When you ask me your seven questions, I am allowed to LIE about at most one of the answers. What seven questions can you ask to determine n?

Alice Bob Alice Bob Alice

i was driving on a highway recently for one hour at a constant and very special speed. what was so special about it? the number of cars i passed was the same as the number of cars that passed me! your speed must have been the mean of the speeds of the cars on the road. or was it the median?

MEAN, MEDIAN,
file:///C|/Documents%20and%20Settings/Administrator/My%20Documents/My%20Notepad/Jayesh%20placment/hard.shtml (15 of 69)8/7/2008 3:24:49 PM

[ wu :: riddles(hard) ]

NEITHER

Bob

these two are often confused. maybe it's neither? we'll have to think about this.

Was Alice's speed the mean, median, or neither? Note: Assume that any car on the road drives at a constant nonzero speed of s miles per hour, where s is a positive inte- ger. And suppose that for each s, the cars driving at speed s are spaced uniformly, with d(s) cars per mile, d(s) being an integer. And because each mile looks the same as any other by the uniformity hypothesis, we can take mean and median to refer to the set of cars in a fixed one-mile segment, the half-open interval [M, M+1), at some instant.

In an LCD display some numbers, when viewed upside-down, are images of other numbers. For example,

UPSIDE-DOWN LCD DISPLAY

1995 becomes 5661. The fifth number that can be read upside down is 8, and the 15th is 21, which is 12 when viewed upside-down. What is the millionth number that is meaningful upside-down?

A certain hotel room lock is opened by scanning a key card. In theory, one enters the room by inserting and removing the key card once. In practice, however, the key card is ambiguously labeled, so that either of two orientations might be the correct orientation of the card. In theory, of course, one could just try one orientation, and if it didn't work try the other. In practice, however, the card reader sometimes fails, so that after trying each orientation once, one may still not have gained access to the room. A few more details: (a) Either of the two orientations is equally likely to be cor- rect. (b) The success rate of a correctly oriented card is p with 0 < p < 1. (c) Failure on either side of the card is indistinguishable, so it does not give any information about whether the orientation of the card was correct. (d) An attempted scan takes 1 second to succeed or fail; reversing the orientation also takes 1 second. The last property means that in 3 seconds one could try one orientation 3 times or each orientation once
file:///C|/Documents%20and%20Settings/Administrator/My%20Documents/My%20Notepad/Jayesh%20placment/hard.shtml (16 of 69)8/7/2008 3:24:49 PM

HOTEL KEY CARD

[ wu :: riddles(hard) ]

(using the middle second to flip it over). In either case, of course, one might still be standing in the hall and need to decide what to do next. So what attempt strategy would you use to enter the room? Why? Feel free to consider fixed values of p (p = .5 or p = .9, for example) as special cases. What if p is fixed but unknown?

"Here's the deal. You give me $10. Then I will deal four cards (from a regular 52 card deck), chosen randomly, face down. You get to look at #1 first and decide whether to keep it. If not, Alice look at #2 and decide whether to keep that one. If not look at #3, and decide. If you don't take that, then #4 is your choice. If your chosen value is n, I will pay you $n. Then we can reshuffle

CARD GAME
Bob

the entire deck, you give me another $10, and we can play again, and again, and again." "Hmmm....I need a good strategy to beat you at this game, but I think I can do it."

Help Bob out with a strategy that will win. Note that the cards all have face value with the following exceptions: Ace=1, Jack = 11, Queen = 12, and King = 13.

INFINITE COIN FLIPS

Given a coin with probability p of landing on heads after a flip, what is the probability that the number of heads will ever equal the number of tails assuming an infinite number of flips?

file:///C|/Documents%20and%20Settings/Administrator/My%20Documents/My%20Notepad/Jayesh%20placment/hard.shtml (17 of 69)8/7/2008 3:24:49 PM

[ wu :: riddles(hard) ]

There are two envelopes in front of you each with a non-zero sum of money. You are informed one has twice as much money as the other. You are then allowed to select either envelope and keep the money inside. After you select one and before opening it you are given the option to change your mind and

ENVELOPE GAMBLE I

switch to the other one? You think to yourself that if your envelope has x dollars there is a 50% chance the other one has x/2 dollars and a 50% chance it has 2x dollars. The expected return, you compute, is .5 [.5x + 2x]=1.25x which seems like a favorable gamble. Do you switch and why? Assume you are neither risk averse nor risk prone, in other words you will take any good gamble and avoid any bad one.

You are trapped in a small phone booth shaped room. In the middle of each side of the room there is a hole. In each hole there is a push button that can be in either an off or on setting. You can't see in the holes but you can reach your hands in them and push the buttons. You can't tell by feel whether they are in the on or off position. You may stick your hands in any two holes at the same time and push neither, either, or both of the buttons as you please. Nothing will happen until you remove both hands from the holes. You succeed if you get all the buttons into the same position, after which time you will immediately

BUTTON TRAP ROOM

be released from the room. Unless you escape, after removing your hands the room will spin around, disorienting you so you can't tell which side is which. How can you escape? Now generalize. You are in a room with N sides, each side having a hole with a push button. What is the minimum number of hands you need to escape the trap room? Note: Regarding the original setup with N = 4, the fewest possible turns that I know of is seven.

Four heat-seeking missiles are initially placed at the corners of a square with side length s. Each missile

HEAT SEEKING MISSILES M

flies at a constant speed toward the missile on its left. Describe the path each missile takes until it collides with the rest in the square's center. What is this path's length? Generalize to n missiles on a regular n-gon.

file:///C|/Documents%20and%20Settings/Administrator/My%20Documents/My%20Notepad/Jayesh%20placment/hard.shtml (18 of 69)8/7/2008 3:24:49 PM

[ wu :: riddles(hard) ]

A cubic piece of cheese has been subdivided into 27 subcubes (so that it looks like a Rubik's Cube). A

MOUSE EATING CHEESE CUBES

mouse starts to eat a corner subcube. After eating any given subcube it goes on to another adjacent subcube. Is it possible for the mouse to eat all 27 subcubes and finish with the center cube?

The figure below is a square composed of 21 smaller squares. Each of the 21 smaller squares has a side of integer length and all 21 are different sizes. Find any solution for the size of the 21 smaller squares.

21 SQUARES CPU

Of three men, one man always tells the truth, one always tells lies, and one answers yes or no randomly.

TRUTHS, FALSEHOOD, RANDOMNESS

Each man knows which man is who. You may ask three yes/no question to determine who is who. If you ask the same question to more than one person you must count it as question used for each person whom you ask. What three questions should you ask?

Create the number 24 using only these numbers once each: 3, 3, 7, 7. You may use only the following

24 I

functions: +, -, *, /. This is not a trick question; for example, the answer does not involve a number system other than base 10 and does not allow for decimal points.

file:///C|/Documents%20and%20Settings/Administrator/My%20Documents/My%20Notepad/Jayesh%20placment/hard.shtml (19 of 69)8/7/2008 3:24:49 PM

[ wu :: riddles(hard) ]

The religious keeper of the web page The Premature Death of Rockstars argues that rock stars do not live as long as the general population. He states that the average age at death of rock stars is 36.9 and 75.8

STATISTICS LIES

for the general population. What is wrong with this use of these statistics? This is an illustrated example of lying with statistics.

Thirteen pirates put their treasure in a safe. They decide that the safe should be able to be opened if any majority of pirates agree but not be able to be opened if any minority agree. The pirates don't trust each other so they consult a locksmith. The locksmith puts a specific number of locks on the safe such that

13 PIRATES

every lock must be opened to open the safe. Then he distributes keys to the pirates such that every pirate has some but not all of the keys. Any given lock can have multiple keys but any given key can only open one lock. What is the least number of locks required?

You have 12 identical-looking balls. One of these balls has a different weight from all the others. You also have a two-pan balance for comparing weights. Using the balance in the smallest number of times

12 BALLS

possible, determine which ball has the unique weight, and also determine whether it is heavier or lighter than the others.

I am thinking of a number between 1 and 100. You can guess a number and I will tell you if it is high or low. If you get it right on the first guess I will pay you $5, on the second guess $4, and so on. If you get it right on the sixth guess I will pay you nothing, on the seventh guess you owe me $1, on the eight you owe me $2, and so forth (5 4 3 2 1 0 1 2 3 4 5 4 3 2 1 0 1 2 ...)

GUESS NUMBER FOR MONEY I

Q1) Should you play the game with me? Q2) [Slightly harder and more math] What is your expected return? Note: From interview with Gates/Ballmer about 8-12 years ago. Used to test "the way you think about problems".

file:///C|/Documents%20and%20Settings/Administrator/My%20Documents/My%20Notepad/Jayesh%20placment/hard.shtml (20 of 69)8/7/2008 3:24:49 PM

[ wu :: riddles(hard) ]

You have n bitstrings, each of length k. Define the Hamming distance between two bitstrings as the number of bits on which the strings differ (e.g. the Hamming distance between 0000 and 1100 is 2). Assume the strings have been chosen such that the Hamming distance between *any* two strings is maximized. In other words (but perhaps more confusing words), the minimum Hamming distance of the set has been maximized. Write down a closed form formula which computes the maximum minimum Hamming distance for any given n and k.

Note 1: Here is the notion of Hamming space, summarized in a useful picture:

MAXIMUM MINIMUM HAMMING DISTANCE M, >=P

This is the Hamming space for k=3. Start at a vertex. Change one bit to cross one edge. Change two bits to cross two edges. Change three bits to cross three edges. Note the important difference between Hamming space and the everyday notion of numeric distance. Whereas we would normally say that 100 (4 in binary) and 000 (0 in binary) are 4 units apart, in Hamming space, they are only one unit apart, and very close. Note 2: This is a problem that my research partner and I came across while studying error correcting codes. There must exist a closed-form formula, but we have been unable to derive it or find it. For a second, it seemed like we had it, but then it was wrong. It most likely involves mods and floors and
file:///C|/Documents%20and%20Settings/Administrator/My%20Documents/My%20Notepad/Jayesh%20placment/hard.shtml (21 of 69)8/7/2008 3:24:49 PM

[ wu :: riddles(hard) ]

ceilings. Forum thread: click here

The professor for class Logic 315 says on Friday: "We're going to have a surprise quiz next week, but I'm not telling you what day... if you can figure out what day it will be on, I'll cancel the quiz." The students get together and decide that the quiz can't be on Friday, as if the quiz doesn't happen by Thursday, it'll be obvious the quiz is on Friday. Similarly, the quiz can't be on Thursday, because we know it won't be on Friday, and if the quiz doesn't happen by Wednesday, it'll be obvious it's on Thursday (because it can't be on Friday). Same thing for Wednesday, Tuesday and Monday. So it can't be on ANY day, so there's no quiz next week!" They tell the professor, who smiles and says, "Well, nice to see you're thinking about it."

POP QUIZ >=P


On Tuesday, the professor gives the quiz, totally unexpected! What's the flaw in the students' thinking?

Note: (Update 1/28/2003 2:40AM) There are many possible solutions, but I have included the >=P symbol because supposedly the math community has not agreed on an official explanation. I don't fully understand why this is so. Forum thread: click here

file:///C|/Documents%20and%20Settings/Administrator/My%20Documents/My%20Notepad/Jayesh%20placment/hard.shtml (22 of 69)8/7/2008 3:24:49 PM

[ wu :: riddles(hard) ]

Three players enter a room and a red or blue hat is placed on each person's head. The color of each hat is determined by a coin toss, with the outcome of one coin toss having no effect on the others. Each person can see the other players' hats but not his own. No communication of any sort is allowed, except for an initial strategy session before the game begins.

SIMULTANEOUS HAT COLOR GUESSING

Once they have had a chance to look at the other hats, the players must simultaneously guess the color of their own hats or pass. The group shares a hypothetical $3 million prize if at least one player guesses correctly and no players guess incorrectly. The same game can be played with any number of players. The general problem is to find a strategy for the group that maximizes its chances of winning the prize. Hint: See the maximum minimum Hamming distance riddle for some powerful ideas.

a corporate business man has three cubes on his office desk. every month, he arranges the cubes so that the front faces show the current month's three letter abbreviation (e.g. JAN FEB MAR APR MAY JUN JUL AUG SEP OCT NOV DEC }. what letters are on the faces of each cube?

CALENDAR CUBES II

Note 1: This might be insanely difficult. Just remember to be creative. Special thanks to Bruce Preston for sending this to me, along with a wonderful GIF of his solution! Note 2: Although the month abbreviations I have typed above are all capitalized, you may have to be more flexible with your fonts to construct a working solution.

file:///C|/Documents%20and%20Settings/Administrator/My%20Documents/My%20Notepad/Jayesh%20placment/hard.shtml (23 of 69)8/7/2008 3:24:49 PM

[ wu :: riddles(hard) ]

There are three omniscient gods sitting in a chamber: Past, Present and Future. They are all truthful, but with the following caveat: Present answers the question currently being asked, Past answers the last question asked in their chamber, and Future answers the next question which will be asked in their chamber. Despite their manipulation of which question to answer, each still answers immediately as if answering the question currently being asked. Furthermore, the gods answer in a language in which "yes" and "no" are replaced by "da" and "ya", but you do not know which is which. You only know that their answers are consistent amongst themselves. With three questions, determine which god is which.

Note 1: (standard) Because of possible time conflicts, you must determine your questions ahead of time, rather than based on previous answers. You are, however, allowed to choose who you ask each of your three questions to dynamically, and scoping is also dynamic (e.g. the pronoun "you" in a question will always refer to the person you choose to ask the question to, not a predetermined person). No selfreferential questions (e.g. "is this question true iff ..."). No time related questions (e.g., "if the answer to my second question was 'no', then... otherwise ...") are permissible, as this could lead to paradoxes within the space-time continuum). Finally, note that if you ask Past your first question or Future your last question, the answer will give you no additional information because you do not know what the last or next questions are! Note 2: (specific) Because of possible time conflicts, you must determine your questions ahead of time, rather than based on previous answers. However, you are still allowed to choose who you ask each of your three questions to dynamically. Scoping is also dynamic; e.g. the pronoun "you" in a question will always refer to the person to whom you are currently asking a question, not a predetermined person). No time related questions (e.g., "if the answer to my second question was 'no', then X otherwise Y") are permissible, as this could lead to paradoxes within the space-time continuum). Finally, note that if you ask Past your first question or Future you last question, the answer will give you no additional information because you do not know what the last or next questions are!! Note 3: This awesome riddle was actually designed by the contributor, Eric Yeh! E-mail him feedback: click here

PAST, PRESENT, FUTURE

file:///C|/Documents%20and%20Settings/Administrator/My%20Documents/My%20Notepad/Jayesh%20placment/hard.shtml (24 of 69)8/7/2008 3:24:49 PM

[ wu :: riddles(hard) ]

You can start flipping a coin, and at any time claim a prize in cents equal to the fraction of flips that came

COIN FLIP GAME WORTH I

up heads. So, if you stop playing after getting 4 heads in 5 flips, you earn 80 cents. How much is the game worth?

I have a distribution over the Reals which you do not know. I choose two numbers from it, and write them inside envelopes. You are given one of the envelopes, and allowed to see the number inside it.

ENVELOPE GAMBLE II

Then, you are given the option to switch envelopes once. After you settle on an envelope, you win the amount inside your envelope, and you pay the amount inside the other envelope. Can you win money playing this game, with a strategy independent of my distribution?

Create the number 24 using only these numbers once each: 2, 3, 10, 10. You may use only the following

24 II

functions: +, -, *, /. This is not a trick question; for example, the answer does not involve a number system other than base 10 and does not allow for decimal points.

Create the number 24 using only these numbers once each: 1, 3, 4, 6. You may use only the following

24 III

functions: +, -, *, /. This is not a trick question; for example, the answer does not involve a number system other than base 10 and does not allow for decimal points.

An enemy submarine is somewhere on the number line (consider only integers for this problem). It is moving at some rate (again, integral units per minute). You know neither its position nor its velocity.

SINK THE SUB

You can launch a torpedo each minute at any integer on the number line. If the the submarine is there, you hit it and it sinks. You have all the time and torpedoes you want. You must sink this enemy sub devise a strategy that is guaranteed to eventually hit the enemy sub.

file:///C|/Documents%20and%20Settings/Administrator/My%20Documents/My%20Notepad/Jayesh%20placment/hard.shtml (25 of 69)8/7/2008 3:24:49 PM

[ wu :: riddles(hard) ]

willywutang is lonely and needs a wife. So he flips through a phone book and plans to date exactly N

BACHELOR'S DILEMMA M

women at random, none of whom he knows anything about beforehand. To expedite the process, after going on just one date with a woman, he must immediately decide whether or not she is "the one"; this decision is irreversible. Devise an algorithm willy should use to maximize his chances of choosing the best woman in the set. Convince willy that your algorithm is optimal.

A thousand wires hang on a very high tower, so high that you cannot see what tip belongs with what bottom. This is something you are interested in knowing. You have a battery and a light bulb which will light up if two wires connect it to the battery with appropriate polarity (i.e. the battery and bulb each have two contact points, and one of each is + and the other side is -). Wires may be tied together to form

1000 WIRES

longer wires, and you can see the bulb light up even if you are on the opposite side of the tower. Since the tower is so high, you want to minimize the number of times you have to climb up and/or down the staircase, regardless of how much you have to do while you are at the top or bottom. What is the minimum number of traversals required?

You have a checkerboard which extends infinitely in all four directions. One half-plane is covered with checkers on all the black squares. You can jump pieces using normal checker jumps, but must remove the pieces that are jumped. How many rows into the uncovered half-plane can you get a checker? Repeat the problem where all squares are covered, and jumps are vertical and horizontal instead of diagonal as in standard checkers.

INFINITE CHECKERBOARD

file:///C|/Documents%20and%20Settings/Administrator/My%20Documents/My%20Notepad/Jayesh%20placment/hard.shtml (26 of 69)8/7/2008 3:24:49 PM

[ wu :: riddles(hard) ]

In complex analysis, an entire function is defined as a function which is infinitely differentiable at every point in C (for example: constants, polynomials, e^x, etc.). Picard's Theorem says that every nonconstant entire function f misses at most one point (i.e. f(C) = C or C-{x0}). For example, every nonconstant polynomial hits every point, and e^x misses only 0. Now consider the function f(x) = e^(e^x). Since e^x is entire, f is also entire by the chain rule. But it misses 0 since the base e^y misses 0, and it misses 1 since the top e^x misses 0 so that e^(e^x) misses e^0 = 1. But by Picard's Theorem there can be only one missing point, so the two missing points must be the same. Therefore, 0 = 1.

PICARD'S THEOREM PROOF THAT 0 = 1 M

Alice and Bob are going to play a game, with the following rules: Alice picks a probability p, 0 <= p < 0.5 Bob takes any finite number of counters B. Alice takes any finite number of counters A.

COUNTER GAME

These happen in sequence, so Bob chooses B knowing p, and Alice chooses A knowing p and B. A series of rounds are then played. Each round, either Bob gives Alice a counter (probability p) or Alice gives Bob a counter (probability 1-p). The game terminates when one player is out of counters, and that player is the loser. Whom does this game favor? Analyze and discuss probabilities.

A duck is in a circular pond with a menacing cat outside. The cat runs four times as fast as the duck can

DUCK IN THE POND

swim, but cannot enter the water. Can the duck get to the perimeter of the pond without the cat being on top of him?

file:///C|/Documents%20and%20Settings/Administrator/My%20Documents/My%20Notepad/Jayesh%20placment/hard.shtml (27 of 69)8/7/2008 3:24:49 PM

[ wu :: riddles(hard) ]

A lion and a lion tamer are enclosed within a circular cage. If they move at the same speed but are both

LION AND TAMER

restricted by the cage, can the lion catch the lion tamer? (Represent the cage by a circle, and the lion and lion tamer as two point masses within it.)

Someone claims to have invented a Universal Truth Machine (UTM), a machine that takes a proposition as input, and returns "true", "false", or "undecidable" as output. Example: input "2 + 2 = 4" output "true" "false" "undecidable"

UNIVERSAL TRUTH MACHINE

"0 + 2 = 4" "this proposition is false"

Devise a true proposition that the UTM will claim to be false, thereby disproving the inventor's claim. Note: Revised 8/8/2002 4:39PM.

( From Raymond Smullyan's book What is the Name of This Book? ) In WitNoTB?, Smullyan introduces his Knights, Knaves, and Normals. Knights always tell the truth, Knaves always lie, and Normals can do either. In the subchapter titled "How to Marry a King's Daughter", Smullyan tells the story of a king who wants his daughter to marry a nice normal Normal, not one of those goody-goody Knights or devious scoundrel Knaves. Smullyan asks the following questions (easy puzzles): How can you make a true statement that will convince the King you are a Normal? How can you make a false statement that will convince the King you are a Normal? How can you make a statement that will convince the King you are a Normal, but he won't know whether it's true or false? Then Smullyan tells the story of a different King, one who is not so kindly disposed towards Normals. In fact, he sees them as wishy-washy girly-men, unreliable and untrustworthy. A Knight will always be true,
file:///C|/Documents%20and%20Settings/Administrator/My%20Documents/My%20Notepad/Jayesh%20placment/hard.shtml (28 of 69)8/7/2008 3:24:49 PM

[ wu :: riddles(hard) ]

SMULLYAN WAS WRONG

and a Knave is wholly reliable as long as you remember to believe the exact opposite of what he says, while a Normal will deceive you when you least expect it. He wants his daughter to marry anyone but a Normal. Smullyan asks: how many statements must you make to convince the King? Smullyan's answer was: no statements you make can ever convince the King you are no Normal. Since a Normal can say anything, any statement you make could be made by a Normal. There is no way to prove your abnormality to the King. Smullyan was wrong! There is a subtle flaw in his argument, and in fact it is quite possible for a nonNormal to prove it. So the puzzle for you is (harder puzzles): How can you, in one statement, convince the King you are a Knight? How can you, in one statement, convince the King you are a Knave? How can you, in one statement, convince the King you are not a Normal, but leave him unable to deduce whether you are a Knight or a Knave? Note: Problem presentation by Jonathan Haas.

At a movie theater, the manager announces that they will give a free ticket to the first person in line

BIRTHDAY LINE M

whose birthday is the same as someone who has already bought a ticket. You have the option of getting in line at any time. Assuming that you don't know anyone else's birthday, that birthdays are distributed randomly throughout the year, etc., what position in line gives you the greatest chance of being the first duplicate birthday?

A particle is travelling from point A to point B. These two points are separated by distance D. Assume that the initial velocity of the particle is zero. Given that the particle never increases its acceleration along its

PARTICLE TIME

journey, and given that the particle arrives at point B with speed V, what is the longest time that the particle can take to arrive at B?

file:///C|/Documents%20and%20Settings/Administrator/My%20Documents/My%20Notepad/Jayesh%20placment/hard.shtml (29 of 69)8/7/2008 3:24:49 PM

[ wu :: riddles(hard) ]

There are three omniscient gods sitting in a chamber: GibberKnight, GibberKnave, and GibberKnexus, the gods of the knights, knaves, and knexuses of Gibberland. Knights always answer the truth, knaves always lie, and knexuses always answer the XOR of what the knight and knave would answer. Unfortunately, the language spoken in Gibberland is so unintelligible that not only do you not know which words correspond to "yes" and "no", but you don't even know what the two words that represent them are! All you know is that there is only one word for each. With only three questions, determine which god is which.

THE GODS OF GIBBERLAND

Note 1: What follows are standard rules that are generally assumed unless otherwise noted. The gods only answer yes/no questions. Each god answers in the single word of their language as appropriate to the question; i.e. each god always gives one of only two possible responses, one affirmative and one negative (e.g. they would always answer "Yes" rather than "That would be true"). Each question asked must be addressed to a single specific god; asking one question to all the gods would constitute three questions. Asking a single god multiple questions is permissible. The question you choose to ask and the god you choose to address may be dynamically chosen based on the answers to previous questions. No self-referential questions (e.g. "is this question true iff ..."). Note 2: Because of possible loop conflicts, you may not ask any questions regarding how a knexus would answer. Note 3: Designed by the contributor, Eric Yeh! E-mail him feedback: click here.

file:///C|/Documents%20and%20Settings/Administrator/My%20Documents/My%20Notepad/Jayesh%20placment/hard.shtml (30 of 69)8/7/2008 3:24:49 PM

[ wu :: riddles(hard) ]

The town of Friendville has an interesting property. Given any two people in the town, they either know eachother or they don't. If they don't know eachother, then they can be introduced to eachother. One single introduction will work for both people. That is, "Tom this is Phil, Phil this is Tom" counts as one introduction.

INTRODUCTIONS ALL AROUND

The other interesting property that this town has, is that if any group of n people get together, the number of introductions that must be made in order that everyone in the group knows everyone else is at most n-1. Problem: Prove that the town can be divided into two groups (A and B) such that everyone in group A knows each other, and everyone in group B knows each other.

There are two houses of parliament in the land of Orange Milano. Each member of parliament has at most

3 ENEMIES

3 enemies. PROVE that all the members can be placed in a house such that each member will have at most one enemy in the same house.

MEAN DISTANCE TWO POINTS

What is the mean distance between two random points on a unit square?

This is a magic trick performed by two magicians, Alice and Bob, with one shuffled deck of N unique cards. (Nothing is mentioned about suits: you may consider these cards to be simply enumerated from 1 to N.) Alice asks a member of the audience, Carol, to randomly select 5 cards out of a deck. Carol then returns her chosen 5 cards to Alice. After looking at the 5 cards, Alice picks one of the 5 cards and gives it back to Carol. Alice then arranges the other four cards in some way, and gives them to Bob in a neat, facedown pile. Bob examines these 4 cards and determines what card is in Carol's hand (the missing 5th card). Carol is astonished! What is the largest number of cards N that the deck can contain before the trick is no longer performable? Prove it.
q

5 CARD MAGIC TRICK REDUX

How specifically do you execute the trick on a deck of maximal size N?

file:///C|/Documents%20and%20Settings/Administrator/My%20Documents/My%20Notepad/Jayesh%20placment/hard.shtml (31 of 69)8/7/2008 3:24:49 PM

[ wu :: riddles(hard) ]

Note 1: There's no secretive message communication in the solution, like encoded speech or ninja hand signals or ESP or whatever ... the only communication between the two magicians is encoded in the 4 cards transferred from Alice to Bob. Think of these magicians as mathematicians. Note 2: Essentially, this is the same 5 card magic trick, but now I am challenging you to get away with it using a deck larger than 52 playing cards.

100 prisoners in solitary cells. There's a central living room with two light bulbs. The initial states of these light bulbs is unknown. No prisoner can see these light bulbs from his or her own cell. Every now and then, whenever he feels like it, the warden picks a prisoner at random, and that prisoner enters the central living room. While there, the prisoner can toggle the bulbs if he or she wishes. Also, the prisoner has the option of asserting the claim that all 100 prisoners have been to the living room. If this assertion is false (that is, some prisoners still haven't been to the living room), all 100 prisoners will be executed. However, if it is indeed true, all prisoners are set free. Thus, the assertion should only be made if the prisoner is 100% certain of its validity. Before the warden begins this sick game, the prisoners are allowed to get together one night in the

100 PRISONERS AND TWO LIGHT BULBS

cafeteria to discuss a plan. What plan should they agree on, so that eventually, someone will make a correct assertion? (Alternatively phrased: Devise an algorithm that has a 100% chance of success, optimizing for the expected number of warden choices until success.) After that, try generalizing to N light bulbs (disclaimer: I don't know where this N bulb thing leads).

Note: Summary of differences between this riddle and the notorious "100 prisoners and one light bulb": two light bulbs initial state of bulbs is unknown warden chooses whenever he feels like it, rather than once a day.

file:///C|/Documents%20and%20Settings/Administrator/My%20Documents/My%20Notepad/Jayesh%20placment/hard.shtml (32 of 69)8/7/2008 3:24:49 PM

[ wu :: riddles(hard) ]

There are three omniscient gods sitting in a chamber: AND, OR, and XOR. They all answer the truth, but they all apply their namesake operations in one of the following ways: BACKWARD: The operator is applied to ALL of the questions that have been asked THUS FAR. UNIVERSAL: The operator is applied to ALL of the questions (you have to pick your questions beforehand).
q

SELFLESS: The operator is applied to ALL of the questions NOT asked to them (you pick your questions beforehand).

For each of the 3 cases, determine with proof the minimum number of questions that will allow you to identify which god is which.

THE OPERATORS OF BOOLANIA

Note 1: (Standard; rules that are generally assumed unless otherwise noted.) The gods only answer yes/ no questions. Each god answers in the single word of their language as appropriate to the question; i.e. each god always gives one of only two possible responses, one affirmative and one negative (e.g. they would always answer "Yes" rather than "That would be true"). Each question asked must be addressed to a single specific god; asking one question to all the gods would constitute three questions. Asking a single god multiple questions is permissible. The question you choose to ask and the god you choose to address may be dynamically chosen based on the answers to previous questions. Note 2: (Specific) Because of possible time conflicts, you must determine your questions ahead of time, rather than based on previous answers. However, you are still allowed to choose who you ask each of your three questions to dynamically. Scoping is also dynamic; e.g. the pronoun "you" in a question will always refer to the person to whom you are currently asking a question, not a predetermined person. No time related questions (e.g., "if the answer to my second question was 'no', then X otherwise Y") are permissible, as this could lead to paradoxes within the space-time continuum. Note 3: Another gem designed by Eric Yeh. E-mail him feedback: click here.

EVERY DIE FACE

You roll a k-sided die n times. What is the probability that each of the k faces appeared at least once?

file:///C|/Documents%20and%20Settings/Administrator/My%20Documents/My%20Notepad/Jayesh%20placment/hard.shtml (33 of 69)8/7/2008 3:24:49 PM

[ wu :: riddles(hard) ]

The set of numbers {9, 99, 999, 9999, ...} has some interesting properties. One of these has to do with factorization. Take any number n that isn't divisible by 2 or by 5. You will be able to find at least one number in the set that is divisible by n. Furthermore, you won't need to look beyond the first n numbers

STRINGS OF NINES

in the set. Prove it.

There are three puzzlers in the puzzle forum: A Newbie, a Senior Riddler, and an Uberpuzzler. All three are honest, but can only give answers to the best of their knowledge. Newbies are confused creatures. Until their fifteenth posts, they are only able to make random responses! * Senior Riddlers have great powers of perception, but are not yet infallible. In fact, they have been known to give incorrect responses up to once per day!** They are always able to deduce the identities of their fellow puzzlers, but have no access to individual thoughts or the future. Uberpuzzlers are omniscient beings who are your greatest allies in the Puzzle Forum!!! Not only do they always know and tell the truth, but they also have a special power of Influence! Uberpuzzlers are able to telepathically communicate with anyone else in the room, and thereby clarify the person's thoughts so that he knows the answer to the current question. However, they must do so BEFORE the other person chooses an answer, or it may be too late. The Uberpuzzler can exert Influence arbitrarily often. However, before each session in the Puzzle Forum, he randomly chooses a limit for how many times he will apply his special power (i.e. how helpful he will be). The result is a secret; all you know is that the limit is always a positive integer. Furthermore, an Uberpuzzler only uses his power of Influence in a very specific way. First, he determines the goal of the puzzler (which may, for example, be to identify all three puzzlers in five questions). Then he will determine the strategy of the puzzler (in this case, it would be a binary tree of questions, with each node determining a question and the person who should be asked; i.e. a fully dynamic set of questions and answerers). Finally, IF POSSIBLE UNDER HIS CHOSEN INFLUENCE LIMITATION, he will determine circumstances (in this case, questions following certain reply paths such as TFF or T or TFFTF) for each possible situation (in this case, permutations of the three people) under which he would apply
file:///C|/Documents%20and%20Settings/Administrator/My%20Documents/My%20Notepad/Jayesh%20placment/hard.shtml (34 of 69)8/7/2008 3:24:49 PM

[ wu :: riddles(hard) ]

THE PUZZLE FORUM

his Influence, such that over all situations, the final set of responses would be distinct relative to the situation. (Thus, he employs a strategy defined as a mapping f : S3 x {(T|F)*} -> {0,1}, which can be interpreted as follows: for each ordering of the puzzlers "sigma" (a permutation in S3, e.g. USN) and set of responses "s" (a string in (T|F)*, e.g. TFTF), he will exert Influence on the question following a response of s in the permutation sigma iff f(sigma,s) = 1. The Uberpuzzler may choose ANY mapping f such that the strings of all possible responses will be disjoint for the six orderings of the puzzlers -- IF POSSIBLE given the limitations. Note that this is different from saying that the puzzler chooses when the Uberpuzzler applies Influence! In general, the Uberpuzzler may have more than one way to make the answer sets disjoint, but the puzzler will not know which one he is employing!!!) Determine with proof the minimum number of questions which will allow you to identify which puzzler is which. [* The Newbie does not make any new posts during questioning. ;) ] [** It is assumed that the question asking session takes place during the confines of one day.] Note: Writing credits to Eric Yeh! The titles in this riddle are taken after various member rankings you can earn in our wonderful riddle forum. E-mail him feedback: click here

file:///C|/Documents%20and%20Settings/Administrator/My%20Documents/My%20Notepad/Jayesh%20placment/hard.shtml (35 of 69)8/7/2008 3:24:49 PM

[ wu :: riddles(hard) ]

Willywutang recently took a personality test ... he failed :) Actually, the test was set up so he couldn't fail. There were four personality "colors", and the objective was to decide which color was strongest in him. There were three test sections, and in each section, he had to rank the four colors from 1 (weakest) to 4 (strongest). At the end of the test, he summed up the rankings from each section to get his personality "profile". The minimum he could score on any color was therefore 3, and the maximum was 12. After he knew how he scored for each color, the test asked him to color in a circle (with crayons) in proportion to his color scores. Willywutang complained that he didn't have a protractor to measure out the exact angles. (I wonder what this says about his personality?) Willywutang then remarked loudly that the test makers could have divided up the circle beforehand (piechart style) and then he would only have to color between the lines. The natural question is, of course: What is the minimum number of divisions that must be made so that the circle can be colored to accurately represent any possible proportion of colors? How are these divisions arranged? Note: Writing credits to James Fingas!

WILLY'S TRUE COLORS

You have a deck of 52 cards - for convenience, number them 1 through 52. You cut the cards into two equal halves and shuffle them perfectly. That is, the cards were in the order 1,2,3,...,52 and now they are 1,27,2,28,...,26,52. Let's call this a perfect in-shuffle. If you repeat this in-shuffling process, will the cards ever return to their initial ordering? If so, how many in-shuffles will it take?

SHUFFLING CARDS
How does the solution change if you have a deck of 64 cards, or 10, or in general, n cards? For odd integer values of n, in-shuffling will take 1,2,3,...,n to 1,(n+3)/2,2,(n+5)/2,...,n,(n+1)/2. For example, when n=5, the first shuffle in-yields 1,4,2,5,3. What if you do out-shuffles instead? (27,1,28,2,...,52,26)

file:///C|/Documents%20and%20Settings/Administrator/My%20Documents/My%20Notepad/Jayesh%20placment/hard.shtml (36 of 69)8/7/2008 3:24:49 PM

[ wu :: riddles(hard) ]

The diabolical Two-Face has acquired an atomic bomb, and is setting it to detonate somewhere in Gotham City in 2 days. Two-Face plans to deactivate the bomb only if the mayor agrees to pay him 2 bazillion dollars. The bomb can be deactivated with a 2,000 digit passcode that only Two-Face will know. In signature style, Two-Face wants exactly half of the passcode digits to be "2"s, and the other half to be "1"s. Two-Face anticipates that Batman, the world's greatest detective, will probably find the bomb somehow. To reduce chances of Batman cracking the passcode and foiling the plan, Two-Face wants to choose a passcode equally at random, and without sacrificing the aforementioned symmetry. One way of doing this would be to just conjure "2"s and "1"s randomly off the top of his head, while making sure that in the end, the total number of "2"s equals the total number of "1"s. However, Two-Face doesn't trust his mind to be truly random. It sure would be nice if he had a computer or calculator to generate random numbers, but there are no such devices nearby. So Two-Face once again pulls out his fair-sided coin, the elegant tool he uses to make all decisions in life. Using only his coin, how can Two-Face construct a satisfactory passcode equally at random? And at least how long / how many flips will the construction take? Note: Writing credits to William Wu =)

TWO-FACE BOMB

A Bright Star is a sphere, not just a point, that emits light in all directions from every point on its surface. A Dark Star is an opaque sphere whose dull black surface reflects no light. Dark Stars come in all sizes. Your assignment is to get and position a few of them around a given Bright Star in such a way as will absorb all its light, thus rendering it invisible from afar in every direction. What is the minimum number of Dark Stars required to carry out this task, and why?

BRIGHT AND DARK STARS

Note 1: Brought to Math H90 at UC Berkeley by Jason Behrstock in 1995. Note 2: I've decided to cross-list this riddle in the putnam section. The accessibility of the beautiful premise makes it suitable here, but the solution may seem more appropriate for a mathematical audience. Couldn't make up my mind.

file:///C|/Documents%20and%20Settings/Administrator/My%20Documents/My%20Notepad/Jayesh%20placment/hard.shtml (37 of 69)8/7/2008 3:24:49 PM

[ wu :: riddles(hard) ]

Borromean Circles are three perfect circles (perhaps of different radii) made of wire and so artfully linked in space that they cannot be separated; however, if any one circle were removed, the others would fall apart. Example diagram:

BORROMEAN CIRCLES
Prove that perfect Borromean Circles are actually impossible to construct.

Note: (Useless historical information) Borromean circles were the symbol of the Borromean League founded in 1586 to reimpose Catholicism over Protestant areas of Switzerland, which was almost destroyed by consequent prolonged warfare. The League named itself after a controversial but widely admired St. Carlo Borromeo (1538-84, canonized in 1610), Cardinal and Archbishop of Milan, whose influential family had been established on the four little Borromean islands in Lake Maggiore, which runs North-East across the border between Italy and the Swiss canton of Ticino.

file:///C|/Documents%20and%20Settings/Administrator/My%20Documents/My%20Notepad/Jayesh%20placment/hard.shtml (38 of 69)8/7/2008 3:24:49 PM

[ wu :: riddles(hard) ]

The Moonstone, a fabulous gem, has been stolen again. Witnesses saw one person steal it, but cannot describe him (or her) at all, nor has any physical evidence been found. Police suspect some one of the three most notorious jewel thieves may be responsible and have all three under close surveillance, including telephoto lenses and paraboloidal microphones, but lack a reason to arrest any of them. Each suspect, denying involvement in the theft, has expressed his curiosity: Who took the Moonstone this time? Later the three were observed dining together. Each one had occasion to leave their table and visit

MOONSTONE

the lavatory; during his absence the two at the table flipped a (presumably fair) coin, hiding its face from the absentee (and from the surveillance team) but not from each other. After the third suspect returned, all three winked at each other and dispersed silently. The surveillance team saw only the back of one suspects head when he winked, so his wink was not recorded. The team says all three suspects know now whether one of them stole the gem but, if he did, the other two do not know which one did; and the team does not know whether one did. How was all this information communicated?

When equations z = z(y) and y = y(x) can be solved for x = x(z) satisfying z = z(y(x(z))), then 1 = dz/dz = (dz/dy)(dy/dx)(dx/dz) as if all the d...'s were parts of fractions that cancelled each other out. But in a different situation when an equation f(x, y, z) = 0 can be solved for any one of the variables x, y, z as a function of the other two satisfying, for example, f(x, y, z(x,y)) = 0, then (@z/@y)(@y/@x)(@x/@z) = -1 where "@" denotes the funky partial derivative symbol that looks like a reversed "6". Prove this last equation and explain why the "@"s don't cancel out the way the d...'s did.

DERIVATIVE CANCELLATION M

file:///C|/Documents%20and%20Settings/Administrator/My%20Documents/My%20Notepad/Jayesh%20placment/hard.shtml (39 of 69)8/7/2008 3:24:49 PM

[ wu :: riddles(hard) ]

3 overlapping circles are drawn to cut the plane into 7 finite regions, each with 3 circular arcs as its boundary. 7 coins are placed, 1 in each region, all Heads-Up. Then a game is played, consisting of a sequence of coin flipping steps. At each step a circle is chosen, and one of the following two operations is performed upon all coins in it: Operation A: Turn every coin Heads-Up. Operation B: Reverse every coin.

HEADS-UP

The object of the game is to put the coin in the central (inside-most) region Heads-Down, and all other coins Heads-Up. Is this objective accomplishable? Hint: Consider what each kind of operation will do the following predicate concerning an odd number of coins: "Every circle has an odd number of conis Heads-Down."

Alice and Bob play a game. Starting with Alice, they alternate in selecting digits for a 6-digit decimal number UVWXYZ that they construct from left to right. Alice chooses U, then Bob chooses V, then Alice chooses W, and so on. No digit can be repeated. Alice wins if UVWXYZ is not a prime. Can Alice always win?

UVWXYZ M

Update 11/4/2002 8:42PM: There was a typo: I had written "Alice wins if UWXYZ is not a prime". Thanks [SWF].

file:///C|/Documents%20and%20Settings/Administrator/My%20Documents/My%20Notepad/Jayesh%20placment/hard.shtml (40 of 69)8/7/2008 3:24:49 PM

[ wu :: riddles(hard) ]

Consider the following process: 1. Pick a positive integer. 2. Square each of digits and add the squares together, to get a new number.

HAPPY AND SAD NUMBERS >=P

3. Repeat steps 2-3 with the new number. "Happy numbers" are numbers destined to reduce to one. "Sad numbers" are those that get stuck in an infinite loop of distinct numbers; for example, the loop { 89, 145, 42, 20, 4, 16, 37, 58, 89, 145, ... }. In general, what kinds of numbers are happy, and what kinds are sad? Give a closed-form expression for which integers are happy, and which integers are sad.

Player A has n+1 coins, while player B has n coins. Both players throw all of their coins simultaneously

2N+1 COINS

and observe the number that come up heads. Assuming all the coins are unbiased, what is the probability that A obtains more heads than B?

A solid cube 20x20x20 is built out of bricks each 2x2x1 . All bricks are laid with their faces parallel to the

BRICK PIERCING

cubes faces, though bricks need not be laid flat. Prove that at least one straight line perpendicular to a face of the cube pierces its interior but no bricks interior.

HIGH BUILDINGS AND STREET VENDORS >=P

Consider a country in which people live in high buildings and buy things from street vendors, using some form of money. Each building is equipped with a pulley in the eaves over which runs a long rope with a basket on it, operable by either party. Is there a system by which a resident and a vendor who don't trust each other can conduct trade? Note that either party may plan to play the opposite role with someone else later.

file:///C|/Documents%20and%20Settings/Administrator/My%20Documents/My%20Notepad/Jayesh%20placment/hard.shtml (41 of 69)8/7/2008 3:24:49 PM

[ wu :: riddles(hard) ]

You are given a Rubik's cube in a randomly chosen initial position. Denote a "move" as a 90 degree rotation of a face. "God's Algorithm" is the name of an algorithm which solves a Rubik's cube in the fewest number of moves. 1. Determine a lower bound for the number of moves God's Algorithm requires. 2. Determine an upper bound. 3. Can you devise a small, simple algorithm to solve a Rubik's cube in the minimum number of moves? (e.g. not brute-force lookup tables)

Note 1: Forum thread: click here Note 2: What is a Rubik's cube? It is the most famous puzzle in history. Enro Rubik invented it in 1974, and already about one-eighth of the world's population has had its hands on it. The Rubik's cube consists of 27 smaller cubes. Initially, all faces of the cube are the same color. You can rotate 9-block sections of the cube to break this pattern, as illustrated by the image below. Afterwards, the objective is to return the Rubik's cube to its initial state of monochromatic faces. Given that you didn't know the steps taken to randomize the cube faces, this is surprisingly difficult to do.

RUBIK'S CUBE >=P

Enro Rubik was a lecturer in the Department of Interior Design at the Academy of Applied Arts and Crafts in Budapest. While giving lectures, he liked to illustrate his ideas with actual models made from wood, cardboard, plastic, and other common materials. He created the cube as an experiment in aesthetic geometric forms, and to help students with spatial visualization. Rubik himself did not expect the process of returning the cube to its initial state to be so difficult. Eventually the Cube caught on as an international phenomenon. Today it is a common household item, and the subject of many mathematical papers.

file:///C|/Documents%20and%20Settings/Administrator/My%20Documents/My%20Notepad/Jayesh%20placment/hard.shtml (42 of 69)8/7/2008 3:24:49 PM

[ wu :: riddles(hard) ]

A corporate business man has two cubes on his office desk. Every day he arranges both cubes so that the

CALENDAR CUBES III

front faces show the current day of the month in Roman numerals. What numbers are on the faces of the cubes to allow this? You can have as few (including zero) or as many numerals on a single face as you wish.

When the Prince of Trianglia (an island where all intersections are Ys, and no roads lead to dead ends) returned from his voyage of discovery, he issued two edicts: 1. Because he spent a month trapped by a washed-out road, new roads are to be built so that there is more than one route to every portion of Trianglia. 2. Because dirt roads are tiresome and boring, all roads will be paved with either red, yellow, or blue bricks (each road having only one color). The color of each road should be chosen so that every intersection is the meeting of roads of all three colors.

TRIANGLIA II

Can the Trianglians assign a color to each of the roads to satisfy the second edict? If so, how? A quote from the "Fun Facts about Trianglia" Tourist brochure: "Trianglians are very superstitious about having things over their heads when they are travelling. There are no overpasses or tunnels anywhere in Trianglia."

Note: Writing credits to Paul Sinclair!

file:///C|/Documents%20and%20Settings/Administrator/My%20Documents/My%20Notepad/Jayesh%20placment/hard.shtml (43 of 69)8/7/2008 3:24:49 PM

[ wu :: riddles(hard) ]

We have a discrete time signal x[n] that is bandlimited to pi/3. In other words, if X() is the discretetime fourier transform (DTFT) of x[n], then: for pi/3 < < pi, |X()| = 0. In the process of retrieving x[n] from a hard disk, one sequence value has become corrupted, resulting in a corrupted signal y[n]. The graphs below show the real and imaginary parts of the DTFT of y[n].

DATA CORRUPTION M

1. By examining the above graphs, estimate n0, the index of the sequence value that has been corrupted. Explain. 2. If y[n0] = x[n0] + c, estimate c. Explain your reasoning.
file:///C|/Documents%20and%20Settings/Administrator/My%20Documents/My%20Notepad/Jayesh%20placment/hard.shtml (44 of 69)8/7/2008 3:24:49 PM

[ wu :: riddles(hard) ]

Note 1: Requires some knowledge of the discrete-time fourier transform. A pretty cool puzzle nevertheless. Note 2: Writing credits to Dr. Avideh Zakhor, UCB EE 123 Fall 2001. No, it's not from my homework this week ;)

Willywutang had been travelling for nearly two days when he came to the fork in the road. Willy knew it was inevitable, but he was prepared. Willy knew from the various puzzles he had solved that one path led to the City of Safety, and the other path led to the City of Cannibals. There should also be two people waiting at the intersection, who, willingly or not, would tell him how to get to the City of Safety ... Slouched against a tree trunk near the intersection was a scruffy stranger. Willy stopped in his tracks and stared at the man, very worried. He was certain to be from one of the two cities, but which one? Where was the second person? The man returned Willy's gaze, saying nothing. There was a tense silence, broken only when the mysterious stranger finally spoke up.

FORK IN THE ROAD III

"If you're wonderin' where the other guy is, he's home sick today. You see, he's from the City of Cannibals, and someone came through here with a bad cold. You're lucky that I'm here, from the City of Safety!" Willy was too much on his guard to be reassured by the scruffy man's story. Perhaps it was true, and perhaps it wasn't. Willy also knew the rules about getting directions--he could ask just one yes/no question. Is there any way Willy could find out which path led to the City of Safety?

Note: Writing credits to James Fingas!

file:///C|/Documents%20and%20Settings/Administrator/My%20Documents/My%20Notepad/Jayesh%20placment/hard.shtml (45 of 69)8/7/2008 3:24:49 PM

[ wu :: riddles(hard) ]

Three criminals just robbed a bank and go back to their hideout. They put the money behind a high tech security door. There are 3 locks on the door, each activated/deactivated by a button next to it. All locks are originally deactivated, and once a lock is activated it is impossible to tell whether it is activated or not. The three criminals want to work out a system so that any two of them can access the money but a

THREE LOCKS

single criminal cannot. The 2 criminals accessing the money must be assured that all locks are deactivated, otherwise an alarm will sound, and built-in lasers will shoot them. Also, each criminal may only give information about which locks he toggled to one other criminal. Figure out how their system will work.

Here is the story of three unhappy people: Angelica, Bernardo, and Cameron. Angelica and Cameron have been friends since childhood. Cameron is hopelessly in love with Angelica, but Angelica has always thought of Cameron as "just a friend". During high school, a new boy, Bernardo, moves into the area. Bernardo immediately catches the attention of Angelica, who falls head-over-heels for him. However, Bernardo is not interested in women-he is strongly attracted to Cameron. Cameron, of course, is jealous of Bernardo, because he has stolen Angelica's love away. Angelica is angry at Cameron, because she feels that Bernardo's lack of attention to her is Cameron's fault. Bernardo is jealous of Angelica, who recieves all of Cameron's attention. What are these three to do? (Please, this is a rhetorical question--there is no need to answer it) One day, you meet them, all together in a chat room. They are all using nicknames: Uberkewl, Vaxxipaxx, and Willywutang (in no particular order). Because they are so jealous and screwed-up, each of them will only answer a question truthfully only if: 1. one of your last two questions was to their sweetheart, and 2. your last question wasn't to the person with whom they are upset. Otherwise, they will answer spitefully--giving you the answer that will confuse you the most. Your task is to figure out who is who. Is it possible? If so, how many questions might you have to ask?

THE LOVE TRIANGLE

Note: Writing credits to James Fingas!


file:///C|/Documents%20and%20Settings/Administrator/My%20Documents/My%20Notepad/Jayesh%20placment/hard.shtml (46 of 69)8/7/2008 3:24:49 PM

[ wu :: riddles(hard) ]

There are three families, each with two sons and two daughters. In how many ways can the sons all be monogamously matched with the daughters? Generalize to M families, each with N sons and N daughters.

MARRIAGE MATCHINGS
Note: Assume you can't marry within your own family, and all persons are heterosexual. Also, no one has a sex change.

In the Hemlock Grove retirement villa, four seniors are discovering the joys and sorrows of a "love square". Amelia, Bertrand, Claire, and Donald have gotten themselves mixed up in a little more than they can handle. Amelia has fallen for Bertrand, with his youthfully craggy face and his deep gravely voice. Bertrand does not realize this, and secretly admires Claire, with her youthful temperament and derring-do. Claire is oblivious to Bertrand's feelings, and is instead obsessed with Donald and his mysterious past. Donald doesn't know that Claire is interested in him, but he has the hots for Amelia. Of course, Bertrand realizes that Claire likes Donald, and has secretly vowed to get Donald on the bad side of the cleaning staff. Donald shares Bertrand's emnity, since his beloved Amelia is bewitched by the old git. He has been trying to get a bird to build its nest over top of Bertrand's patio door for nearly a year now. Amelia recognizes the glint in Bertrand's eye when he looks at Claire, and she is determined to sabotage

THE LOVE SQUARE

Claire's next candlelight supper by replacing her baking powder with powdered salt. Claire is likewise jealous of Amelia, that slut, and she plans to run her over on the sidewalk with her electric three-wheeled buggy. One day, you meet all four of these unhappy individuals in a chat room, going under the nicknames UsUxOrZ, VampireBob, WussMania, and Xanadu. Because they are all slightly crazy (they are in love after all), each person will only answer a question correctly if:

file:///C|/Documents%20and%20Settings/Administrator/My%20Documents/My%20Notepad/Jayesh%20placment/hard.shtml (47 of 69)8/7/2008 3:24:49 PM

[ wu :: riddles(hard) ]
q

one of your last two questions was asked of their sweetheart, and your last question was not to their enemy.

Otherwise, they will tell you whatever they feel like telling you. Is there any way that you can deduce the identities of these four individuals? How many questions must you ask to determine who is who?

Note: Writing credits to James Fingas!

A man is stuck in a box. There are no doors, no windows, no openings of any kind. Inside with him is a round table and a mirror. How does he get out?

STUCK IN A BOX

Big Hint: This is perhaps an unfair riddle. You'll need a cute sense of humor to solve it without this hint. Think wordplay.

Imagine an infinitely extending grid of squares. Between every two adjacent intersection points is a resistor of 1 ohm. Compute the resistance between two adjacent intersection points.

INFINITE RESISTOR NETWORK

Note: Anyone know how to solve this problem by comparing resistance to commute times of random walks on graphs? Huv + Hvu = 2mRuv, where Huv is the hitting time between vertices u and v, and Ruv is the resistance between u and v. We should be able to show that the ratio of commute time to edges converges to a stable value as the grid size approaches infinity. E-mail me at wwu at ocf.berkeley.edu if you have an idea.

file:///C|/Documents%20and%20Settings/Administrator/My%20Documents/My%20Notepad/Jayesh%20placment/hard.shtml (48 of 69)8/7/2008 3:24:49 PM

[ wu :: riddles(hard) ]

A great advance has occured in the field of artificial intelligence. A computer has been programmed to predict your actions, and has proven itself able to do so with amazing accuracy. You have put it through many difficult and devious tests, yet it has always been correct in its predictions. A certain multibillionaire with a twisted sense of humor has decided to reward you for your great discovery. He take you into a room with two boxes and tells you that box A contains $10,000 while box B either contains nothing, or a certified check for 1 billion dollars. You will be allowed to open exactly one box and keep it contents. The catch is this: Earlier, your program was asked what you would do. If the computer predicted you would open box A, then the 1 billion dollar check was placed in box B. If the computer predicted you would open box B, the box will be empty. Which box should you open? Note that box B was prepared before you enter the room.

NEWCOMB'S DILEMMA

Note: From the pages of Martin Gardner. "Newcomb" refers to William A. Newcomb, a physicist at Lawrence Livermore Laboratory, who came up with the puzzle in 1960.

Find different arrangements of the 13 pieces below to form two different figures: a (non-square) rectangle and a square. Each shape must use all 13 pieces as part of the shape. Click here for a larger figure more suitable for printing and cutting.

ARRANGE THE 13 PIECES

file:///C|/Documents%20and%20Settings/Administrator/My%20Documents/My%20Notepad/Jayesh%20placment/hard.shtml (49 of 69)8/7/2008 3:24:49 PM

[ wu :: riddles(hard) ]

Note: Writing credits to [SWF].

Turkey Sandwich was worried about an upcoming test in Discrete Mathematics and was finding it hard to get to sleep. Turkey awoke early in the morning, aroused by devilish laughter, only to see an impish looking homunculus sitting at the bottom of the bed next to a seemingly infinite pile of chips. Hello Turkey it said, would you like to play a little game? This pile contains 43546758343209876 chips and the bottom chip represents your immortal soul. The rules are quite simple. The first player takes some chips, but not all of them. After that we take it in turns to take some chips.

TAKE THE LAST CHIP

The only rule now is that a player cannot take more in their turn than the previous player took. The winner is the player who takes the last chip. If I win I get to keep your soul and if you win, you get an A in the test. Would you like to go first or second? This seemed a reasonable bet to Turkey. Can you give Turkey a strategy for playing no matter how many chips there are?

Note: Writing credits to Alan Frieze and Danny Sleator.

Is it possible for two checker players, playing cooperatively, to king all twelve of both players' checkers? Keep in mind the following rules of checkers: 1. The initial checkers setup is to use only the black squares of a chessboard, with red checkers on the 12 North-most squares, and black checkers on the 12 South-most squares. 2. Players alternate turns, and cannot pass their turn 3. If a player can "jump" an opposing player's piece, he must do so. This is where all the difficulty arises. 4. If a player cannot jump, then he must move one of his checkers diagonally. All non-kinged checkers can only move forward. 5. When a checker gets to the opposing end of the board, it is "kinged" and can now move backwards as well as forwards.

COOPERATIVE CHECKERS

file:///C|/Documents%20and%20Settings/Administrator/My%20Documents/My%20Notepad/Jayesh%20placment/hard.shtml (50 of 69)8/7/2008 3:24:49 PM

[ wu :: riddles(hard) ]

Note: Writing credits to James Fingas, who doesn't know the answer either :)

1/19/2003 1:56AM

Two cylinders of equal radius are intersected at right angles as shown at left. Find the volume of the intersection between the two cylinders, without using calculus! A 3D picture of the intersection is shown at right.

CROSSED CYLINDERS

Hint (medium hint - exactly which high school formulae you need): 1) Area of circle = pi * radius2, and 2) Volume of sphere = (4/3) * pi * radius3 Note: Solved by the mathematician Archimedes (287 B.C. - 212 B.C.), waaay before calculus came around!

1/19/2003 1:56AM Here's the puzzle: How do you make sense of this puzzle? I'd like to salvage the following interesting puzzle such that it is solvable, but I don't know how to -- or even if the answer is supposed to be that it's not solvable. Exact quote: "There is a Man A, who claims to know Tamil Language. You dont know Tamil language so you can not test him. You hire another Man B, who knows Tamil Language, to test this. Both A and B knows your language too. Though you dont trust both of these guys still you want to test that A knows B. Also you know that A and B dont know any other language too. How will you test man A knows Tamil ? "

file:///C|/Documents%20and%20Settings/Administrator/My%20Documents/My%20Notepad/Jayesh%20placment/hard.shtml (51 of 69)8/7/2008 3:24:49 PM

[ wu :: riddles(hard) ]

I think it's supposed to be a good puzzle because the page I found it on had it next to sink the sub and single-file hat execution, as examples of problems the webmaster discusses with his CS faculty. Problem is, I don't know what entails not trusting B. Does it mean B will translate our English queries reliably, but won't reliably tell us if A responds in Tamil properly? I e-mailed the author for clarification and his answer was meaningless, and then I e-mailed again and he stopped responding (I think he's sick of me). I won't reprint his e-mail address here. Perhaps we can figure out what is necessary to fix the problem?

LANGUAGE PROFICIENCY VERIFICATION >=P

Note 1: I believe the riddle is supposed to say "you want to test that A knows Tamil" instead of "you want to test that A knows B." Note 2: [redPepper] from the riddle forum offers the following amusing answer: "I found the correct riddle's wording. Here it is: " 'There is a Man A who claims to know English language. He's a CS student and likes logic riddles. You like riddles too so you ask him for one. Unfortunately the riddle is worded poorly, which is a problem for a logic riddle where every detail is important. English is obviously not Man A's native language. You can hire another Man B who knows Man A's language and also proper English, to translate the riddle. Though you don't know what is Man A's language. Maybe it's Tamil. You can communicate with Man A in English through e-mail but you can only ask one question, after which any new e-mail will be ignored. How will you gather a correctly worded riddle?' "

1/19/2003 1:56AM A taut rubber band connects a wall to the back side of a toy racecar 1 meter away. Starting at time t=0, the racecar drives away from the wall at 1 meter per second, stretching but never breaking the rubber band. At the same time, an ant resting on the rubber band near the wall starts to move toward the racecar at 1 centimeter per second. Does the ant ever reach the racecar? If so, in exactly how much time? Why?

file:///C|/Documents%20and%20Settings/Administrator/My%20Documents/My%20Notepad/Jayesh%20placment/hard.shtml (52 of 69)8/7/2008 3:24:49 PM

[ wu :: riddles(hard) ]

WALKING ON A STRETCHING RUBBER BAND

Note 1: Treat the ant as a dot, like in geometry. Its initial position at t=0- is 0 distance away from the wall. Note 2: Assume that the angle of incidence between the rubber band and the wall never changes. (In other words, this is all happening along one dimension.) Note 3: Does this remind you of something related to cosmology ("the study of the physical universe considered as a totality of phenomena in time and space")? Note 4: Forum thread (spoilers galore, but interesting physics discussions): click here

1/19/2003 1:56AM A bacterium asexually reproduces k children for the next generation according to a probability distribution P(X = k), where X is a random variable that returns the number of children it has between 0 and +infinity. After reproducing, a bacterium dies.

BACTERIAL EXTINCTION

1. What is the probability that the bacterial colony lasts forever? 2. In general, what statistical property should an offspring distribution have to ensure immortality?

Note: A complete answer to the 2nd question should discuss two cases, one of which is a special case.

2/3/2003 3:30PM

TOPOLOGICAL
file:///C|/Documents%20and%20Settings/Administrator/My%20Documents/My%20Notepad/Jayesh%20placment/hard.shtml (53 of 69)8/7/2008 3:24:49 PM

[ wu :: riddles(hard) ]

RINGS

Imagine the object above in the figure to the left made from perfectly elastic material. Can you transform it so as to unlink the two rings as in the figure on the right? One possible way is to cut one ring, move the other ring through the gap, and rejoin the the first ring exactly as it was. That would be a legitimate topological transformation. However, it is also possible to transform the first shape into the second without any cutting, simply by manipulating the objects in the appropriate manner (stretching, bending, but not breaking). Can you see how to do it?

2/5/2003 3:57AM A prisoner is awaiting execution by firing squad. A row of N robotic soldiers stands nearby. These robotic soldiers can be in a finite number of states (they are all finite state machines). A clock provides a sequence of time steps: 0, 1, 2, etc.. At each step, the state of each soldier is given by a transition function. This function takes three arguments as input: the soldier's previous state, the previous state of the soldier to the right, and the previous state of the soldier to the left. The transition function must be identical for all soldiers except for the soldiers at the ends of the row, whom could have different transition functions. One of the soldiers at one end of the row is deemed the general of the squad. Initially at time 0, the general is in state qinit, whereas all other soliders are in state qsleep. The objective is to get all soldiers to enter the state qbang simultaneously for the first time, thereby riddling the prisoner with N bullets. Design state machines and transition functions for the robotic soldiers which will accomplish this

FIRING SQUAD SYNCHRONIZATION

objective, given that the number of states for a soldier cannot depend on N. What is the running time of your system in terms of N?

file:///C|/Documents%20and%20Settings/Administrator/My%20Documents/My%20Notepad/Jayesh%20placment/hard.shtml (54 of 69)8/7/2008 3:24:49 PM

[ wu :: riddles(hard) ]

Hint (achievable running time): If your machines are sufficiently clever, you can achieve O(N) time! There's even a solution that takes exactly 2N - 2 time steps.

Note 1: If we did not specify "for the first time", a trivial solution would be to have all soldiers keep firing, and eventually they will all be in state qbang. Note 2: In a sense, the heart of this problem is that no soldier knows what N is. Note 3: Problem originally posed by J. Myhill in 1957. Note 4: This problem describes something called a cellular automaton. The soldiers are like "cells", and how a cell is updated depends on the state of nearby cells. Perhaps the most well-known cellular automaton is John Conway's Game of Life.

2/5/2003 4:00AM The number of ascents in a permutation is the number of times when one number is greater than the one

ASCENTS IN PERMUTATIONS

directly preceding it. Examples: [1,2,3,4,5] has 4 ascents in it, (2>1, 3>2, 4>3, 5>4), [1,4,3,2,5] has 2 ascents in it, (4>1, 5>2), and [5,4,3,2,1] has none. Create a program to count the number of permutations of n distinct numbers with exactly r ascents in it.

2/5/2003 4:00AM 1. Discrete-Case N cities are located at integer coordinates on a Cartesian grid. Your task is to lay down roads along the grid such that all the cities are connected, and the total amount of road you lay down is minimized. Devise an algorithm to solve this problem in general. 2. Continuous-Case
file:///C|/Documents%20and%20Settings/Administrator/My%20Documents/My%20Notepad/Jayesh%20placment/hard.shtml (55 of 69)8/7/2008 3:24:49 PM

[ wu :: riddles(hard) ]

N cities are located at real coordinates on the Cartesian plane. Your task is to lay down roads such that all the cities are connected, and the total amount of road you lay down is minimized. Devise an algorithm to solve this problem in general. 3. N-Dimensional Case N cities are located at real coordinates in an N-dimensional world. Your task is to lay down roads such that all the cities are connected, and the total amount of road you lay down is minimized. Devise an algorithm to solve this problem in general.

ROAD LAYOUT

Note 1: (potentially unnecessary clarification) I use the modifier "connected" in the graph theory sense, meaning there exists a path from one city to any other city. In the Discrete-Case, when I say "along the grid", I mean that roads can only travel horizontally or vertically. A road network is comprised of line segments, and the endpoints of any line segment should lie on integer coordinates. Also, note that the problem does not restrict the endpoints of a line segment to the locations of the cities. So it will often be beneficial to invent intermediate points where roads should intersect. Finally, of course the more efficient your algorithm is, the better. Note 2: John Leen, who did graduate research on bubble surfaces, told me what the problem is officially called. The discrete case is called the Rectilinear Steiner Tree problem, and the continuous case is the Euclidean Steiner Tree problem. Formally, you are given a weighted graph in which a subset of the vertices are identified as terminals, and you want to find a minimum-weight connected subgraph that includes all terminals. In general, finding a Steiner Tree is NP-hard. Note 3: [Icarus] from the forum gives us some fascinating facts: 'The continuous case can also be approached by experimental means. Set up a map on a board and place pegs at the location of each town. Place a second thin board on top to form a "peg sandwich". Dip the contraption in a good bubble solution an after when you pull it out, cut holes in the thin board to allow any trapped air to escape. The remaining bubble film between the pegs should follow the lines of a road system that is a local minimum (that is, small variations will all be longer, but it is possible that a major variation would be shorter).'

file:///C|/Documents%20and%20Settings/Administrator/My%20Documents/My%20Notepad/Jayesh%20placment/hard.shtml (56 of 69)8/7/2008 3:24:49 PM

[ wu :: riddles(hard) ]

2/2/2003 3:31AM A hacker is attacking a computer network. Each computer on the network is connected to various other computers. The hacker releases a worm on a source computer. When a worm infects a computer, that computer can propagate a copy of the worm to a connected computer at a rate of once per second. Suppose the hacker knows the layout of the network; i.e., he knows which computers are connected to which. How should the worm proceed to infect the whole network as quickly as possible? Also, if possible, describe an algorithm for computing the minimum time till total network infection. (Admittedly a computer science background will help here, but it is not absolutely necessary.)

WORM PROPAGATION >=P

Note 1: Writing credits to Yosen Lin. Note 2: (Updated 4/7/2003 1:11AM) It appears that Yosen has rediscovered a very difficult problem, the minimal gossip time problem. None of us know of a good solution to this besides brute-force, which would take really, really long. There is some tradeoff involved between graph bushiness and branch depth, which leads me to suspect that the problem is NP-hard. If anyone has some clever ideas, please email me.

3/9/2003 4:48AM Given a bunch of (x,y) coordinates that form the vertices of one non-self-intersecting polygon, how would you go about finding the area of that polygon? What is the most clever / elegant solution you can devise? The desired answer is remarkably simple. Note: Notice I did not say the polygon has to be regular or convex. A regular polygon has all sides of equal length. A convex polygon is a polygon such that all line segments formed between any two vertices must lie inside the polygon boundaries.

POLYGON AREA

file:///C|/Documents%20and%20Settings/Administrator/My%20Documents/My%20Notepad/Jayesh%20placment/hard.shtml (57 of 69)8/7/2008 3:24:49 PM

[ wu :: riddles(hard) ]

Let us distinguish Towns from Cities. Airlines avoid cancelling service between cities for fear of losing busy airports gate assignments to competitors. However, in hard times, airlines do cancel nonstop services between towns, and sometimes cancel nonstop services between towns and cities. The FAA insists that every town be connected to at least one city by nonstop airline service. Every town is so connected now, and some towns to more than one city. Suppose that, for every collection of towns, the number of towns in the collection does not exceed the number of cities each connected currently by nonstop airline service to at least one town in the collection. Show how airlines can cancel some citytown nonstop flight services leaving no city connected to more than one town and yet every town connected to just one city, barely satisfying the FAA, without adding any new service to the flightschedule. Explain why your chosen process works.

HARD TIMES FOR AIRLINES

4/7/2003 12:17AM Suppose you have an infinite number of small balls, all of which have been uniquely numbered from 1 upwards. What is more, you have an infinitely large bucket, by which I mean it can be made as large as necessary. You decide to fill the bucket by throwing in all the balls, in order. Starting from 1, every minute you throw in two balls. But every minute, an impish pixie takes one ball back out. He always extracts the lowest-numbered ball in the bucket. For example : 1st minute: You throw in Ball 1 and Ball 2. Pixie extracts Ball 1.

2nd minute: You throw in Ball 3 and Ball 4. Pixie extracts Ball 2.

3rd minute: You throw in Ball 5 and Ball 6.

IMPISH PIXIE

Pixie extracts Ball 3.

and so on...

QUESTION: After an infinite amount of time has elapsed, how many balls are in the bucket?
file:///C|/Documents%20and%20Settings/Administrator/My%20Documents/My%20Notepad/Jayesh%20placment/hard.shtml (58 of 69)8/7/2008 3:24:49 PM

[ wu :: riddles(hard) ]

Argument 1: There is an infinite number of balls in the bucket. After 1 minute there is 1 ball. After 2 minutes there are 2 balls. After 3 minutes there are 3 balls, etc. Argument 2: There are no balls in the bucket. If there are some balls in the bucket, what is the lowestnumbered ball? It can't be Ball 1; that was extracted after 1 minute. Similarly, it can't be Ball 2; that was extracted after 2 minutes. It can't be Ball 3; that was extracted after 3 minutes, etc. If the phrase 'after an infinite amount of time has elapsed' bothers you, then we can change the problem so that the 1st put-in-and-take-out operation is completed in 1/2 minute, the 2nd operation is completed in 1/4 minute, the 3rd in 1/8 minute, and so on. Now you can ask the question after 60 seconds, and "infinite time" is not longer an issue.

4/7/2003 12:17AM A property of ellipsoids is that if the inside surface is a perfect reflector, a ray of light originating at one focus will reflect off of the ellipsoid and pass through the other focus. In the figure below FC is a portion of ellipse with foci A and B. DE is a portion of a larger ellipse also having foci A and B. Arcs CD and EF are part of the circle with center B. Points A, C, and D lie on the same line and A, E, and F are also collinear.

ELLIPSOID POWER GENERATION

file:///C|/Documents%20and%20Settings/Administrator/My%20Documents/My%20Notepad/Jayesh%20placment/hard.shtml (59 of 69)8/7/2008 3:24:49 PM

[ wu :: riddles(hard) ]

Think of a perfect reflector made by revolving this shape about line AB to form an axisymmetric shape. Any photon originating at A must strike one of the ellipsoid surfaces and will be reflected toward B. For photons orginating at B, some strike the ellipsoids and reflect toward A, but some also strike the spherical portion of the reflector and come right back to B. Take two small identical objects of the same temperature and place one at A and the other at B inside the reflector. Also the rest of the space inside the reflector is vacuum. Objects emit thermal radiation based on temperature in the form of photons. All the radiation from A strikes B. Only a fraction of that emitted from B arrives at A. So with time B will become hotter than A. In theory, this temperature difference can be used to generate power. If this would not work, why not? If it would work, why aren't ellipsoids being used to generate power?

4/7/2003 12:17AM

THREE COLLINEAR INTERSECTIONS

Take any three circles of differing radius in the plane. For each pair of circles, draw two tangent lines that touch both circles and cross each other outside the convex hull of the circles. You now have three intersections, one for each pair of circles. Prove that these three points lie on a line.

4/7/2003 12:17AM A game begins with 1.e4 and ends in the fifth move with knight takes rook mate. Reconstruct the game.

CHESS GAME
file:///C|/Documents%20and%20Settings/Administrator/My%20Documents/My%20Notepad/Jayesh%20placment/hard.shtml (60 of 69)8/7/2008 3:24:49 PM

[ wu :: riddles(hard) ]

RECONSTRUCTION

Note: This puzzle has apparently stumped many smart people, including Garry Kasparov, Anatoly Karpov, Mikhail Botvinnik, and Ken Thompson (a la Unix and C). Major kudos if you solve it yourself!

4/7/2003 12:17AM In a stripped-down version of Conway's game of life, cells are arranged on a square grid. Each cell is either alive or dead. Live cells do not die. Dead cells become alive if two or more of their immediate neighbours are alive. (Neighbours to north, south, east and west.) What is the smallest number of live cells needed in order that these rules lead to an entire N X N square being alive? And how should those cells be arranged? Prove your claims.

"ERIK'S PUZZLE"
In a d-dimensional version of the same game, the rule is that if d neighbours are alive then you come to life. What is the smallest number of live cells needed in order that an entire N X N X ... X N hypercube becomes alive? And how should those live cells be arranged? Prove your claims.

file:///C|/Documents%20and%20Settings/Administrator/My%20Documents/My%20Notepad/Jayesh%20placment/hard.shtml (61 of 69)8/7/2008 3:24:50 PM

[ wu :: riddles(hard) ]

Note: From David MacKay's book Information Theory, Inference, and Learning Algorithms.

4/7/2003 12:17AM 1. What is the area of the smallest ellipse that can be circumscribed around a 3-4-5 triangle? 2. What is the area of the largest ellipse that can be inscribed in a 3-4-5 triangle? Exact answers only. Hint (small): The dimensions of the triangle are not important - they were only given to offer some

STEINER ELLIPSES

perspective to the problem.

Note 1: This one comes from Jakob Steiner, a Swiss geometer in the 19th century. Note 2: A cool quote from the great Martin Gardner about this problem: "... it is one of the best examples I know of a problem that is difficult to solve by calculus or analytic geometry but is ridiculously easy if approached with the right turn of mind and some knowledge of elementary plane and projective geometry." Thanks [Icarus].

4/7/2003 12:17AM The southeast puzzle is played on a semi-infinite chessboard, starting at its northwest (top left) corner. There are three rules: 1. In the starting position, one piece is placed in the northwest-most square, as shown in (a). 2. It is not permitted for more than one piece to be on any given square. 3. At each step, you remove one piece from the board, and replace it with two pieces, one in the square immediately to the East, and one in the the square immediately to the South, as illustrated in (b). Every such step increases the number of pieces on the board by one. After move (b) has been made, either piece may be selected for the next move. (c) shows the outcome of moving the lower piece. At the next move, either the lowest piece or the middle piece of the three may be
file:///C|/Documents%20and%20Settings/Administrator/My%20Documents/My%20Notepad/Jayesh%20placment/hard.shtml (62 of 69)8/7/2008 3:24:50 PM

[ wu :: riddles(hard) ]

selected; the uppermost piece may not be selected, since that would violate rule 2. At move (d) we have selected the middle piece. Now any of the pieces may be moved, except for the leftmost piece.

SOUTHEAST

Puzzle: Is it possible to obtain a position in which all of the ten squares closest to the northwest corner, marked in the rightmost figure (z), are empty? Hint 1: What does this puzzle have to do with data compression? Hint 2: Symbol codes.

Note: From David MacKay's book Information Theory, Inference, and Learning Algorithms.

5/11/2003 11:47PM Below are 10 statements, all either true or false: 1. At least one of statements 9 and 10 is true. 2. This is either the first true statement or the first false stament. 3. There are three consecutive false statements. 4. The difference between the number of the last true statement and the first true statement divides the number which is to be found. 5. The sum of the numbers of the true statements is the number which is to be found. 6. This is not the last true statement. 7. The number of each true statement divides the number which is to be found. 8. The number that is to be found is the percentage of true statements.
file:///C|/Documents%20and%20Settings/Administrator/My%20Documents/My%20Notepad/Jayesh%20placment/hard.shtml (63 of 69)8/7/2008 3:24:50 PM

THE MIRRORED MAZE

[ wu :: riddles(hard) ]

9. The number of divisors of the number that is to be found (apart from 1 and itself) is greater than the sum of the numbers of the true statements. 10. There are no three consecutive true statements. Find the minimum admissible number (which is to be found).

Contributor: Leonid Broukhis

5/27/2003 2:01PM

On April 1st a typist found the hammers of the typewriter resoldered in an arbitrary order, so typing a text resulted in gibberish. The typist decided to type a document in its entirety using this typewriter. Afterwards, if the result does not represent the original text, he will type the result, and so on. Prove that

TYPEWRITER GIBBERISH

the clear text will emerge sooner or later. How many iterations are enough to guarantee that the clear text will appear, if the typewriter has, say, 46 keys? N keys?

Author: Leonid Broukhis

5/27/2003 2:01PM A particle starts at origin O in three-space. Think of the origin as centered in a cube 2 units on a side. One move in this walk sends the particle with equal likelihood to one of the eight corners of the cube. Thus, at every move the particle has a 50-50 chance of moving one unit up or down, one unit east or west, and one unit north or south. If the walk continues forever, find the fraction of particles that return to the origin.

THREE DIMENSIONAL RANDOM WALK

file:///C|/Documents%20and%20Settings/Administrator/My%20Documents/My%20Notepad/Jayesh%20placment/hard.shtml (64 of 69)8/7/2008 3:24:50 PM

[ wu :: riddles(hard) ]

Contributor: Paul Jung

9:00 PM 8/19/2004

TEMPERATURE ANTIPODES

Assume the earth is a perfect sphere. Show that at any given time, there exist two antipodal points with the same temperature.

Contributor: Thomas Cover The expected projected area of a convex set S is 1/4 the surface area of S.

9:00 PM 8/19/2004

CONVEX SET PROJECTION

The following hints outline one possible approach: Hint 1: Think about the expected projection of a single line segment of length L. (this calculation should not be difficult) Hint 2: With your work with respect to Hint 1 in mind, think about the line segments that make up a convex polygon. Consider the projections of a convex polygon.

Contributor: William Wu

9:00 PM 8/19/2004

Call a set of cubes incongruent if they all have different side lengths. Prove that it is impossible to exactly

FILLING A BOX WITH CUBES

fill a rectangular box with incongruent cubes. Note: The phrase "exactly fill" means that there is no space in the box which is not occupied by a cube, and that the cubes themselves should be packed together to form the shape of the rectangular box that envelops them.

file:///C|/Documents%20and%20Settings/Administrator/My%20Documents/My%20Notepad/Jayesh%20placment/hard.shtml (65 of 69)8/7/2008 3:24:50 PM

[ wu :: riddles(hard) ]

Contributor: Icarus

9:00 PM 8/19/2004

There are a couple of ways of defining the 10-adics "A10". Conceptually, the easiest is to just "flip" traditional decimal notation: A 10-adic number has a decimal representation that can continue indefinitely to the left, but must terminate to the right. For example, instead of the infamous 0.999..., in the 10-adics you have ...999.0 = ...999. Addition, subtraction, multiplication, and division are performed exactly as they are for real numbers. Carries that "go on to infinity" are lost. E.g. ...999.3 + 1 = 0.3. For the most part, these operations obey the same rules as they do for real numbers. For instance, addition and multiplication are commutative and associative, and multiplication still distributes over addition. On the other hand, there is no ordering ("a < b" is generally undefined for 10-adics). Thus, while

10-ADIC NUMBERS

irrational 10-adics may appear to be "infinitely big", the concept is without meaning for them. The 10-adics do not require a "unary minus": ...999 + 1 = 0, so -1 = ...999. Some challenges, in increasing difficulty: 1. For any 10-adic number x, describe how to calculate -x. 2. Find 1/7 as a 10-adic number. More generally, for integer N, describe how to calculate 1/N. 3. Show that the 10-adics possess zero-divisors (i.e. non-zero numbers whose product is zero). 4. Show that the 2-adics (same construction, but with base 2) do not possess zero-divisors. (The 2adics form a field.) For what values of N do the N-adics possess zero-divisors?

Author: Paul Sinclair

file:///C|/Documents%20and%20Settings/Administrator/My%20Documents/My%20Notepad/Jayesh%20placment/hard.shtml (66 of 69)8/7/2008 3:24:50 PM

[ wu :: riddles(hard) ]

Contributor: William Wu

9:00 PM 8/19/2004

Prove that there exists a 101 digit string which contains exactly one copy of every possible 2-digit string

DEBRUIJN SEQUENCE

over the alphabet {0,1,2,3,4,5,6,7,8,9}.

Note: Check out forum thread Willywutang and the Number Stamp for hints and solutions.

Contributor: Barukh

9:00 PM 8/19/2004

A magician takes a deck of 36 cards (nine cards of each suit) lying face down. He predicts the suit of the top card, turns it up and puts aside. Then he predicts the suit of the second card, turns it up and puts aside, and so on with the entire deck. The deck for the trick was prepared by his secret assistant who knows the order of the cards. He uses the fact that the cards in the deck are slightly non-symmetric, and so the magician can distinguish between two possible orientations of every card. The assistant is not allowed to alter the order of the cards, but he can choose the orientation. What is the best strategy the magician and the assistant should agree upon in order to maximize the number of *guaranteed* correct predictions?

GUESSING CARDS SUIT

Source: Moscow Mathematical Olympiad.

Contributor: Barukh

9:00 PM 8/19/2004

(Nasty Geometry) Given a segment AB in the plane and a region R, as shown in the figure. It is desired to continue the line AB to the right of R. How may this be done with straightedge alone so that the straightedge never crosses R during the construction?

PROLONG THE LINE


file:///C|/Documents%20and%20Settings/Administrator/My%20Documents/My%20Notepad/Jayesh%20placment/hard.shtml (67 of 69)8/7/2008 3:24:50 PM

[ wu :: riddles(hard) ]

Clarification: The "straightedge" here is a classical Euclidian tool capable of constructing the straight line segment defined by any two arbitrarily chosen points, or extending an existing straight line segment an arbitrary distance in either direction (Thanks rmsgrey for pointing out).

Contributor: William Wu

9:00 PM 8/19/2004

In drawing lowercase English letters a through z uniformly at random, the expected time till you see willywilly is 26^10 + 26^5. In drawing numerals over 0-9 uniformly at random, the expected time till you see 1231231 is 10^7 + 10^4 + 10.

WAITING FOR PATTERNS

In flipping a fair coin, the expected time till you see HTHTTHT is 2^7 + 2^2.

See a pattern? Find it ... then prove it.

Forum thread: For a write-up of my solution, see thread Expected number of throws before n heads in a row.

file:///C|/Documents%20and%20Settings/Administrator/My%20Documents/My%20Notepad/Jayesh%20placment/hard.shtml (68 of 69)8/7/2008 3:24:50 PM

[ wu :: riddles(hard) ]

Thanks to Contributors

David Lau, Siddhartha Doshi, Jiong Shen, Carl Wang, Phoebus Chen, Alan Liu, Hansen Bow, Ernest Zhu, Elaine Lo, Yosen Lin, Don Barkauskas, Katherine Chan, Jasvir Nagra, Tau Beta Pi (TBP), Eta Kappa Nu (HKN), Danny Dulai, His Grace The Duke Of Ankh-Morpork Commander Sir Samuel Vimes, Michael Snyder, Dipan K. Ghosh, Eric Cole, Louis Wainwright, Ben Bardill, Patrick Dreker, Autumn Looijen, Stephen Champailler, Christopher Kings-Lynne, Bart Samwel, Kannan Ramchandran, Nick Yee, Steve Plimpton, Levsen Hendrik, Remco Hartog, [I.M._Smarter_Enyu], Philip Mock, Michael Chang, Jon Meilstrup, Ryan Russell, Matt Young, Jonathan Haas, Geoff Canyon, Peter Surda, Cory Ondrejka, Satish Rao, [gcooper], Ted Powell, Brave Sir Robin, Eric Cole, J. A. H. Hunter, Sean R. Owen, Andrew Glenn, Bruce Preston, Peter Ratiu, Michael Mendelsohn, Rob Mahurin, James Fingas, Bryan Organ, Jeroen Rutten, Stephen Montgomery-Smith, Marko Lukat, Eric Yeh, Nick Hobson, Mike Lawther, [anshil], Richard Feynman, Douglas Hofstaeder, Dacher Keltner, David Mace, [SAS], Matthew Schultheis, John Leen, Andrew Ooi, Folkert Hindriks, Steve Ragle, Daniel Filner, Karl Barrus, Misha Kruk, Keith Lloyd, Dave Minott, Jette Randlov, Eric Winger, Nathan Hellweg, Tom VanCourt, Chris Seaton, Mitchell Morris, Michael Styer, Zameer Andani, Jonathan Blow, Jeff Thompson, Jonathon Duerig, Dan Hanson, Gabriel Sechan, Tom Saxton, [HunterWare], [alsee], James Antill, Tom Barringer, Bart Massey, David Krikorian, Eric Sharkey, [tudorb], Kevin Day, Milan Ramaiya, Robert Merkel, James Jones, Haim Bitner, Adam Barth, Oscar Lazzarino, Damien Fisher, [DrkShadow], Erik Blankendaal, Eric Smith, James Demmel, Jonathan Shewchuk, Alex Harris, Michael Kelley, [Mr. _Martingale], Kaisen Lin, Hakan Yilmaz, Freddy Mercury, Justin Rising, Marko Lukat, William Kahan, Jeremy Randolph, Michael Sinatra, David Messerschmitt, Patrick Masterson, Frederik Bonte, Randy Williams, Pietro K.C., Brett Danaher, Derek Abbott, Ralph Boleslavsky, Rui del-Negro, [college math journal], [amer. math monthly], Spyros Potamianos, Gary Hsieh, [rec.puzzles], Steven Rudich, Matt Lahut, Richard Reti, Paul Sinclair, Tim Mann, [ucb engineering news], Luke Percival, Anwis Das, Mike White, Louise Poon, Jeffrey Wilhelm, Anthony Cammon, [BNC], A.Frieze & D.Sleator, [SWF], Ted Stevens, Frank Wang, Danny P, Patrick Sesulka, [towr], Chi Sum Cheung, Ranjit Jhala, Jacob Scott, David McKay, Eamon Warnock (THUDandBLUNDER), Kozo Morimoto, Abhijit Joshi, Devesh Parekh, Amnon Melzer, Mary Lou, Leonid Brouhkis, Allistair Sinclair, Mark Newheiser, Joc Koelman, Paul Jung, Aryabhatta, Thomas Cover, Barukh, Nootch, Eigenray

William Wu 2002-04: home | intro | easy | medium | hard | microsoft | cs | putnam | cigs | faq | pros | cons | laff | forum | credits

file:///C|/Documents%20and%20Settings/Administrator/My%20Documents/My%20Notepad/Jayesh%20placment/hard.shtml (69 of 69)8/7/2008 3:24:50 PM

You might also like